Anda di halaman 1dari 52

1. A.A.

is a 21-year-old college student who presents in


the clinic with penile discharge and burning on urination. He admits drinking too much last weekend
(about 6 days ago) and to having unprotected intercourse with a girl he met at a party. A.A. is otherwise
healthy with no comorbidities, chronic medications,
or known drug allergies. A physical examination is
performed, and a urethral discharge smear is taken.
The smear reveals moderate gram-negative diplococci and many polymorphonuclear cells. Which is
the most appropriate treatment recommendation for
this patient?
A. Ceftriaxone 250 mg intramuscularly once.
B. Doxycycline 100 mg orally twice daily for
7 days.
C. Ceftriaxone 250 mg intramuscularly once
plus azithromycin 1 g orally once.
D. Ceftriaxone 250 mg intramuscularly once
plus doxycycline 1 g orally once.
1. Answer: C
The patients clinical presentation and presence of gramnegative diplococci from a urethral smear are
consistent with gonococcal infection. Ceftriaxone 250 mg intramuscularly is the drug of choice.
Fluoroquinolones
are not recommended because of increasing resistance.
When treating a patient for gonorrhea, treatment of chlamydia should also be initiated unless the infection is
specifially ruled out because of high rates of coinfection.
Chlamydia can be treated with azithromycin 1 g orally
once or doxycycline 100 mg orally twice daily for 7 days.
==================================
2. B.B. is a 20-year-old woman who presents to her
primary care provider with painful, ulcerative lesions on her labia bilaterally. Her physician makes
the presumptive diagnosis of herpes labialis and requests your opinion on treatment. Which is the most
appropriate therapy for this patients fist episode?
A. Valacyclovir 1 g orally three times daily
for 7 days.

B. Valganciclovir 900 mg orally once daily


for 7 days.
C. Acyclovir 800 mg orally twice daily for 7 days.
D. Acyclovir 400 mg orally three times daily
for 7 days.
2. Answer: D
For fist episodes of herpes labialis, acyclovir 400 mg
orally three times daily is a reasonable fist-line option.
Increasing the acyclovir dose to 800 mg is not necessary,
and the correct valacyclovir dose for HSV is 1 g orally
twice daily. Although valganciclovir is active against
HSV, its spectrum is broader than necessary.
==========================================

3. G.G. is a 49-year-old female renal transplant recipient who presents to the clinic for a routine follow-up.
She has no food or drug allergies. G.G.s allograft is
functioning well, and she has not been treated for
rejection. Because the inflenza season has just begun, which is the most appropriate means of prevention for
this patient?
A. Oseltamivir 75 mg orally once daily for the
duration of the inflenza season.
B. Inactivated inflenza vaccine and oseltamivir
75 mg orally once daily for 2 weeks.
C. Inactivated inflenza vaccine.
D. Live attenuated inflenza vaccine (LAIV)
7. Answer: C
Vaccination is the most effective method for preventing
inflenza . The inactivated vaccine is preferred in this
patient because she is a transplant recipient. The use of
oseltamivir for 2 weeks at the time of inflenza vaccination (to provide protection until immunity is
established)
may be considered if a patient has an inflenza exposure.
======================================
4. H.H. is 62-year-old woman who presents to her primary care provider for an annual follow-up. She
states that she cannot recall ever having chickenpox
or shingles. Which is the best option to prevent herpes zoster in this patient?
A. Obtain varicella zoster virus (VZV)
immunoglobulin G (IgG), and if negative,
give Varivax; if positive, give Zostavax.
B. Give Varivax.
C. Give Zostavax.
D. Give varicella zoster immune

globulin (VariZIG).
. Answer: C
The ACIP guidelines recommend a one-time dose of
Zostavax to all individuals at 60 years or older, regardless of herpes zoster history. It is thus unnecessary to
obtain VZV serologies for this patient. Varivax is recommended as a routine childhood vaccine and may be
given
to certain immunocompromised patients who are VZV
negative. The VariZIG vaccine is recommended only as
PEP in high-risk VZV-negative patients.
==================================
Patient Case
5. J.C. is a 28-year-old woman who presents to her primary care physician because she had unprotected
sex
with a male acquaintance. Although she has no symptoms, she is concerned that she may have developed
an STD because this man is notorious for having multiple partners. J.C. takes oral birth control, although
she admits to frequently missing doses. An examination is done revealing a positive pregnancy test and
an NAAT test positive for both N. gonorrhoeae and C. trachomatis. Which is the best treatment option for
this patient?
A. Levoflxacin 250 mg orally once plus azithromycin 1 g orally once.
B. Cefiime 400 mg orally once plus azithromycin 1 g orally once.
C. Ceftriaxone 250 mg intramuscularly once plus azithromycin 1 g orally once.
D. Ceftriaxone 250 mg intramuscularly once plus doxycycline 100 mg orally twice daily for 7 days.
. Answer: C
The patients NAAT tests confim both gonorrhea and
chlamydia. Fluoroquinolones are not recommended because of resistance, nor should the patient receive
floroquinolones or tetracyclines because of her positive pregnancy test. Cefiime is recommended only if
ceftriaxone
is not available. The most appropriate choice is ceftriaxone 250 mg intramuscularly once plus azithromycin
1g
orally once.
==========================
Patient Case
6. J.F. is a 39-year-old man with HIV (CD4+ count 225 cells/mm3, HIV viral load less than 48 copies/mL)
who
was treated for secondary syphilis because of the presence of a diffuse rash, generalized lymphadenopathy,
and a previous primary genital chancre. His RPR titer was 1:64, and his FTA-ABS was positive. He denies
neurologic or ophthalmic complaints and receives benzathine penicillin G by intramuscular injection once.
One and one-half years later, the patient has an RPR titer of 1:32 and a positive FTA-ABS. A lumbar
puncture
reveals the absence of white blood cells (WBCs), normal levels of glucose and protein, and a negative
VDRL
test. Which is the most appropriate treatment at this time?
A. Benzathine penicillin G 2.4 million units intramuscularly once.

B. Benzathine penicillin G 2.4 million units intramuscularly once weekly for 3 weeks.
C. Aqueous penicillin G 24 million units daily intravenously for 10 days.
D. Doxycycline 100 mg orally twice daily for 28 days
. Answer: B
The patients therapy for syphilis failed because his RPR
titer did not decrease by 4-fold at follow-up. The FTAABS may remain positive after effective treatment. A
lumbar puncture is indicated when the RPR does not
decrease by 4-fold 612 months after appropriate treatment. This patients lumbar puncture was negative for
neurosyphilis, so intravenous penicillin is not indicated.
Doxycycline is an alternative to penicillin for treatment
of latent syphilis but is not recommended for this patient
because he has no penicillin allergy. In this case, benzathine penicillin G 2.4 million units intramuscularly
once
weekly for 3 weeks is the optimal treatment for late latent syphilis.
==================================
7. You evaluate a new HIV-seropositive patient (risk factor: MSM) who refuses to consider the initiation of
ARV drugs and does not meet the indications for opportunistic infection prophylaxis. His laboratory values
are unremarkable. He is hepatitis A antibody negative, hepatitis B surface antigen negative and surface
antibody positive, and hepatitis C antibody negative. The patient cannot recall his last tetanus booster and is
up-to-date on all childhood immunizations. The patient is allergic to penicillin (rash). Which vaccination(s)
is best ?
A. Intranasal inflenza, hepatitis B.
B. Inflenza (intramuscular), Pneumovax (PPSV23).
C. Hepatitis A; inflenza (intramuscular); Tdap; Prevnar (PCV13).
D. Hepatitis A and B plus inflenza (intramuscular), diphtheria and tetanus toxoids and acellular
pertussis (DTaP), Pneumovax (PPSV23).
. Answer: C
To prevent new-onset infections, close attention should
be paid to the immunization histories of all patients
with HIV infection. Patients with HIV may receive
any killed vaccinations, whereas live vaccinations
should be deferred in most cases. Given the overlap in
risk factors associated with HIV and the hepatic diseases, consideration should be given to immunizations
against hepatitis A and B. In this patient, hepatitis B immunization is unwarranted because he is hepatitis B
antibody positive and antigen negative (indicating a history
of vaccination). Consideration should be given to yearly
inflenza immunizations with the killed intramuscular
formulation. Among patients with HIV infection, community-acquired pneumonia may be particularly
serious;
therefore, pneumococcal vaccination should be administered (ideally fist dose with Prevnar [PCV13],
followed
by Pneumovax [PPSV23]). Because this patient does not
recall his last tetanus vaccination, a booster dose using
Tdap should be provided today and followed every 10

years thereafter with a Td booster. Of note, in studies of


adults receiving concomitant Prevnar and inactivated inflenza vaccine, the response to Prevnar was
decreased
compared with the response of those who received the
two vaccines 1 month apart. Until further data are available, it may be prudent to separate Prevnar and
inactivated inflenza vaccines by 1 month.
Infection 2
8.A female patient comes to the clinic with a urinary
tract infection (UTI), and you are asked by the nursing student to explain the patients urinary dipstick
because the student is confused by the results. The
patients results are positive for nitrite, leukocyte esterase, protein, and blood in the urine. Specifially,
the nurse asks, Which urine dipstick result, if positive, is most indicative of a UTI?
A. Nitrite positive.
B. Leukocyte esterase positive.
C. Positive protein.
D. Positive blood.
. Answer: A
Leukocyte esterase, proteinuria, and hematuria are not
specifi for a UTI. Although leukocyte esterase indicates
the presence of WBCs in the urine, it could be a sign
of inflmmation in the urinary tract. Proteinuria and
hematuria could be present in other disease states, also.
Nitrite positive indicates the presence of nitrate-reducing
bacteria such as E. coli; therefore, it would be most
indicative of a UTI.
==============================
9.R.T. is an 18-year-old woman who presents to the
clinic with a 2-day history of urinary frequency and
burning, and she noticed this morning that her urine
was very dark red. She admits having unprotected
sex with her boyfriend of 6 months. Her urinalysis
comes back with the following results: urine hazy,
white blood cell (WBC) count of 10 x 103 cells/mm3,
nitrite positive, leukocyte esterase positive, positive
protein, positive blood, and 103 CFU/mL gram-negative rods. Culture results are pending. Given her
clinical and laboratory presentation, which is the
best therapeutic decision for R.T.?
A. Treat her for gonorrhea and chlamydia.
B. Treat her for acute uncomplicated cystitis plus
gonorrhea/chlamydia.
C. Treatment is not needed because the bacterial
inocula are less than 105 CFU/mL.
D. Treat her for acute uncomplicated cystitis.
. Answer: D

Although she could be at risk of gonorrhea and/or


chlamydia because of having unprotected sex, she does
not have vaginal discharge, and her symptom onset
is within 3 days of sexual contact; the usual symptom
onset in women takes up to 10 days. A Gram stain and
NAAT would need to be performed by a clinician using
a vaginal swab to determine whether she has gonorrhea
or chlamydia before treatment is initiated. Signifiant
bacteriuria traditionally has been defied as bacterial
counts greater than 100,000 (105) CFU/mL of urine.
Many clinicians, however, have challenged this statement
as too general. Indeed, signifiant bacteriuria in patients
with symptoms of a UTI may be defied as greater than
102 organisms per milliliter.
===============================
10.The Infectious Diseases Society of America (IDSA)
and the American Thoracic Society (ATS) physicians recommend using guideline-based protocols
for the management of community-acquired pneumonia (CAP). Which outcome has the most evidence
to support the use of these guidelines?
A. Decrease in mortality.
B. Decrease in floroquinolone resistance.
C. Increase in practitioner compliance.
D. Decrease in intensive care unit (ICU)
Admissions.
Answer: A
Several studies have shown a decrease in mortality with
the introduction of guideline-based protocols. A 5-year
study of 28,700 patients with pneumonia admitted
with a guideline protocol showed a 30-day mortality
rate that was 3.2% lower with the guideline than in
patients treated concurrently with nonguideline-based
treatment (Dean NC, Silver MP, Bateman KA, et al.
Decreased mortality after implementation of a treatment
guideline for community acquired pneumonia. Am
J Med 2001;110:451-7). Other studies have shown a
decrease in hospitalizations with guideline protocols, but
not specifially on ICU admission.
==============================
11.A 45-year-old man comes to the clinic with a red
rash on his neck that started about a week ago, the
day he flw back from his camping trip in Maine. He
states that although he picked off a few ticks while
he was there, he does not remember any ticks on his
neck. On examination, he has an erythematous rash

with a bulls-eye pattern on the right side of his neck.


He has no other symptoms. His laboratory tests are
positive for Borrelia burgdorferi antibodies, and he
is given a diagnosis of Lyme disease. Which is the
most appropriate management for his disease?
A. Watch and wait to see if more
symptoms develop.
B. Give ceftriaxone 2 g intravenously daily
for 14 days.
C. Give doxycycline 200 mg orally once.
D. Give doxycycline 100 mg orally twice daily
for 14 days.
. Answer: D
This patient presents to the clinic with early Lyme disease,
a classic bulls-eye rash, and positive B. burgdorferi
antibodies. Treatment is imperative to prevent the
development of late Lyme disease. Ceftriaxone
intravenously would not be used for early disease, but for
cardiac or neurological disease. Treatment for 14 days
with doxycycline would be the treatment of choice.
======================================
12.H.J. is a 19-year-old man who returns to the clinic with
worsening nasal congestion, headache, and severe tooth
pain. He has just completed a 10-day course of amoxicillin/clavulanate 2 g/125 mg orally twice daily. H.J.
says that his symptoms got better for a few days but
that they then continued to get worse during the past
week. He states that he took his drugs as prescribed and
has not skipped any days. He has no known drug allergies. Which regimen is the best recommendation for
the
treatment of H.J.s sinusitis?
A. Amoxicillin 1 g three times daily
for 21 days.
B. Azithromycin 500 mg daily for 21 days.
C. Moxiflxacin 400 mg daily for 10 days.
D. Linezolid 600 mg orally twice daily
for 10 days
Answer: C
This patient is not responding to his current regimen
of high-dose amoxicillin/clavulanate. According to the
new guidelines, he should be switched to a respiratory
floroquinolone such as moxiflxacin, and because they
have failed fist-line therapy, treatment would be extended
to 710 days. Amoxicillin is no longer recommended

for sinusitis. Linezolid would not cover the possibility


of infections with gram-negative pathogens such as H.

inflenzae or M. catarrhalis, and its adverse effects and


cost would be issues.
=======================================
13.R.T. is a 13-year-old boy who presents to the pediatricians offie with a 4-day history of severe sore throat
and a temperature of 101F. He states he can barely
swallow because his throat hurts so badly. On physical examination, he weighs 41.2 kg, and his tonsils are
erythematous and swollen. A throat swab is taken, and
the rapid antigen detection test (RADT) comes back
positive for Streptococcus pyogenes. R.T. has no known
drug allergies. Which treatment recommendation is
most appropriate for R.T.?
A. No treatment necessary.
B. Penicillin benzathine 0.6 million units
intramuscularly once.
C. Trimethoprim/sulfamethoxazole 1 doublestrength tablet orally every 12 hours
for 10 days.
D. Penicillin benzathine 1.2 million units
intramuscularly once.
Answer: D
This patient has a positive strep antigen test, so treatment
with antimicrobial therapy is necessary. Penicillin is
the treatment of choice for group A Streptococcus, so
an intramuscular shot of benzathine penicillin would
be appropriate. The patient weighs more than 27 kg,
so the adult dose of 1.2 million units would be needed.
Group A Streptococcus has a high rate of resistance to
trimethoprim/sulfamethoxazole and would not be a good
choice.
=====================================
14.J.K. is a 45-year-old man who presents to the clinic with a 48-hour history of severe diarrhea. J.K.s
medical history includes type 2 diabetes mellitus,
hypertension, and recent surgical drainage of boils.
He has two children younger than 4 years living at
home. He is on day 12 of his 14-day clindamycin
therapy for the treatment of the carbuncles (boils)
he had drained 12 days ago. Physical examination
reveals signifiant improvement in the drained boils,
and stool studies are positive for Clostridium diffi

cile toxins A and B. Which risk factor is most likely

responsible for J.K.s development of C. diffiile


diarrhea?
A. Type 2 diabetes mellitus.
B. Recent surgery.
C. Living with children younger than 5.
D. Current clindamycin treatment.
Answer: D
One of the most modifible risk factors for C. diffiile
diarrhea is the exposure to antimicrobial agents.
Although some antimicrobials may have the impression
of being more associated with C. diffiile, almost all
antimicrobials can be potential risks because of the
changes they can produce on the gut flra. Changes in
gut flra will increase the risk of toxin-producing C.

diffiile. Health care exposure (e.g., recent surgery) can


be a risk, as can immunosuppression or exposure to other
individuals with C. diffiile diarrhea, but these would not
be the most likely causes in this case.
=====================================
15-F.H. is a 62-year-old man who presents to the clinic
with a weeklong history of a blister on the side of his
right foot that has increased in redness and oozing
for the past 24 hours. F.H.s medical history includes
type 2 diabetes mellitus for 25 years, coronary artery disease, hypertension, chronic kidney disease
(creatinine clearance [CrCl] last week measured
22 mL/minute), and amputation of his right middle
figer 3 years ago because of infection. He has no
known drug allergies. Physical examination reveals
a foul-smelling, pus-filed blister with surrounding
erythema on the inside of the right foot with no necrosis. Radiographic studies are negative for osteomyelitis,
and cultures and sensitivities are pending.
F.H. is being sent for wound debridement and outpatient antibiotic therapy. Which regimen is the best
empiric coverage for F.H.s diabetic foot infection?
A. Trimethoprim/sulfamethoxazole 1 doublestrength tablet orally twice daily for 14 days.
B. Levoflxacin 250 mg orally daily for 14 days.
C. Vancomycin 1 g intravenously every 12 hours
for 14 days.
D. Levoflxacin 750 mg orally daily for 14 days
Answer: B
Diabetic foot infections are usually polymicrobial,
so empiric antimicrobial therapy should cover gram
negatives, gram positives, and anaerobes. Levoflxacin

has coverage against most of these organisms and would


be the best choice. The patient has a CrCl of 22 mL/minute;
therefore, levoflxacin at 750 mg daily would be too
high. Vancomycin has mainly gram-positive coverage,
and the 1-g dose every 12 hours might be too aggressive
given this patients renal function. Trimethoprim/
sulfamethoxazole does not offer adequate coverage
for polymicrobial infections. More severe infections
may require broader coverage and/or hospitalization;
however, this patient has a mild infection.
===============================
16. S.O. is an 18-year-old high school senior who comes
to the clinic worried she is going to get sick. Her best
friend just received a diagnosis of meningitis caused
by N. meningitidis . S.O. is an otherwise healthy teenager with no known drug allergies, and she shows no
signs of infection today. She has not yet received her
meningococcal vaccination because she was going
to wait until she went to college. Which would be the
best recommendation for S.O. at this time?
A. Ceftriaxone 1 g intramuscularly once
daily for 14 days.
B. Meningococcal polysaccharide
vaccination only.
C. Rifampin 600 mg orally twice daily
for 4 days.
D. Ciproflxacin 500 mg orally once
Answer: D
This patient has had close contact with someone who
has meningococcal meningitis, and she is unvaccinated;
therefore, she requires prophylaxis. Ceftriaxone could be
used, but the dose should be 125250 mg intramuscularly
once. This patient will require the vaccine at some point,
but not without receiving prophylaxis. Rifampin is a
good choice, but the dose should be 600 mg twice daily
for 2 days. Ciproflxacin 500 mg orally once is fie for
adult prophylaxis and the patient is 18 years old, so she
could receive this regimen.
==================================
Patient Case
17. E.R. is a 22-year-old woman who calls the clinic today with the chief concern of dysuria, and she feels
as if she is getting another UTI. Although it has been 4 months since her most recent UTI, this is the third
time she has had a UTI since she got married 15 months ago. She is tired of having to come to the offie
to be treated for such a simple infection, and she wants to know whether she can do anything to prevent
this from occurring again. Which is the best recommendation to prevent E.R.s having additional recurrent

infections?
A. She is not a candidate for prophylaxis, and ciproflxacin 500 mg orally twice daily for 14 days should
be initiated for a resistant infection.
B. Use prophylactic postcoital therapy with trimethoprim/sulfamethoxazole single-strength tablet.
C. Use prophylactic postcoital therapy with trimethoprim/sulfamethoxazole 1 double-strength tablet
twice daily for 3 days.
D. Use daily prophylaxis with trimethoprim/sulfamethoxazole 1 double-strength tablet daily for
6 months
. Answer: B
Recurrence develops in about 20% of women with
cystitis. If it has been more than 2 weeks since the
last infection, then it is considered a reinfection and
should be treated with an appropriate course of therapy;
therefore, ciproflxacin for 14 days would not be
appropriate. In women who experience symptomatic
reinfections in association with sexual activity, voiding
after intercourse may help prevent infection. In addition,
single-dose prophylactic therapy with Trimethoprim/
sulfamethoxazole ( single-strength tablet) taken
after intercourse considerably reduces the incidence of
recurrent infection. Self-initiated UTI treatment is also
an option in recurrent infections. Long-term prophylaxis
is usually not initiated until the frequency of UTIs is
more than three per year.
=========================================

Patient Case
18. M.J. is an 85-year-old woman whose daughter brings her to the physicians offie because the family has
noticed that she has been sleeping more lately and that she seems very confused. On
physical examination, M.J. is lethargic and not alert and oriented. Her vital signs include temperature of
97.5F, blood pressure 88/55 mm Hg, heart rate 90 beats/minute, and respiratory rate
27 breaths/minute. A chest radiograph taken in the offie reveals a left lower lobe consolidation, and she is
given a diagnosis of CAP. Using the CURB point-of-care patient scoring system,
which statement is the best recommendation for proceeding with the treatment of M.J.s CAP?
A. Treat her as an outpatient for 3 days and reassess.
B. Treat her as an outpatient for 14 days and follow up.
C. Transfer her to the emergency department at the local hospital to be admitted for treatment.
D. Have her transferred to the ICU immediately
. Answer: C
The CURB scoring system is based on a scale of 05,
giving 1 point for each of the following: confusion
caused by pneumonia, urea nitrogen greater than 7
mmol/L, respiratory rate 30 breaths/minute or greater,
blood pressure less than 90 mm Hg systolic or 60 mm
Hg or less diastolic, and age 65 or older. A score of 2 or

greater indicates a need for more intense treatment and


hospitalization. This patient, whose score is 3, should
be admitted to the hospital for treatment. The CURB
score does not determine whether the patient should be
admitted to the ICU.
=====================================
Patient Case
19. R.C. is a 60-year-old woman who presents to the clinic with a 4-day history of increasing productive
cough,
malaise, wheezing, and fever. Her medical history includes type 2 diabetes mellitus for 20 years, congestive
heart failure, chronic kidney disease, and osteoarthritis. On examination, she has a temperature of 102.3F,
respiratory rate 22 breaths/minute, blood pressure 120/78 mm Hg, and heart rate 90 beats/minute. She is 66
inches tall and weighs 90 kg. Her laboratory values are within normal limits except for SCr 3.0 mg/L and
WBC count 18 x 103 cells/mm3. A chest radiograph reveals consolidation in the right lower lobe. She is
given
a diagnosis of CAP. Which regimen is the best empiric option for managing this patients CAP?
A. Levoflxacin 750 mg orally once daily for 10 days.
B. Azithromycin 500 mg orally once on day 1; then 250 mg orally daily for 4 days.
C. Linezolid 600 mg orally twice daily for 10 days.
D. Azithromycin 500 mg orally once on day 1; then 250 mg orally daily for 4 days plus amoxicillin 500
mg orally twice daily for 10 days.]
. Answer: D
According to the IDSA/ATS guidelines, patients with
comorbidities such as diabetes, immunosuppression,
renal failure, and heart failure should be given a
respiratory floroquinolone (levoflxacin, moxiflxacin,
gemiflxacin) or azithromycin or clarithromycin PLUS
high-dose amoxicillin or amoxicillin/clavulanate
because of the risk of drug-resistant S. pneumoniae. This
patient has a calculated CrCl less than 30 mL/minute;
therefore, levoflxacin 750 mg daily would not be the
appropriate dose.
===============================================
Patient Case
20. L.S. is a 35-year-old man who presents to the clinic with a 3-day history of headache, runny nose, nasal
congestion, and tooth pain. He is an otherwise healthy man with no allergies or comorbidities. He is given a
diagnosis of sinusitis. Which intervention is the best treatment recommendation for L.S.?
A. Loratadine 10 mg daily for 10 days.
B. Amoxicillin 1 g orally three times daily for 10 days.
C. Azithromycin 500 mg orally once on day 1; then 250 mg orally daily for 3 days.
D. Oxymetazoline 2 sprays in each nostril every 12 hours for 3 days.
. Answer: D
The primary cause of sinusitis is viral pathogens, and
differentiating between viral and bacterial causes can

be diffiult. However, viral infections usually precede


bacterial sinusitis infections, and viral infections are selflimiting. This patient has had symptoms for only 3
days,
so the diagnosis is most likely viral sinusitis. Therefore,
antibiotic use should be avoided at this time to decrease
the amount of unnecessary antibiotic use and the risk of
resistance. Antihistamines should be avoided in sinusitis
because they can dry out the nasal mucosa. Use of topical
nasal decongestants such as oxymetazoline will help
with the symptoms.
======================================
Patient Case
21. T.R. is a 4-year-old female toddler who presents to the pediatric clinic with a 3-day history of runny
nose,
sore throat, and temperature of 102F. She lives at home with her mother, father, and 11-year-old brother,
and she attends preschool 3 days/week. On physical examination, she is weighs 19 kg, and her tonsils
are erythematous and inflmed. A throat swab is taken, and her RADT comes back negative for group A
Streptococcus. Which is the most appropriate treatment recommendation for T.R.?
A. Penicillin benzathine 0.6 million unit intramuscularly once.
B. Ibuprofen 150 mg (7.5 mL of 100 mg/5 mL of elixir) every 46 hours as needed.
C. Acyclovir 380 mg (20 mg/kg) orally four times daily for 10 days.
D. Trimethoprim/sulfamethoxazole 76/380 mg (trimethoprim 4 mg/kg) orally every 12 hours for 10 days.
Answer: B
This patient probably has viral pharyngitis, given the
negative rapid strep test. Treatment should be supportive
care (e.g., acetaminophen or ibuprofen for pain and
fever as well as plenty of flids). Antiviral therapy is not
indicated for viral pharyngitis because viral pharyngitis
is a self-limiting infection, and antibiotic use would not
be prudent at this time.
===================================
Patient Case
22. M.M. is a 16-year-old adolescent who presents to the clinic with a 24-hour history of a diffuse, itchy
rash in both of her legs. She went to a pool party yesterday and, about 10 hours afterward, noticed
the development of the rash. She has no other symptoms. M.M. is an otherwise healthy teenager
who takes loratadine as needed for allergies. She currently weighs 55 kg. On physical examination,
signifiant fidings are a diffuse erythematous papular follicular rash. She is given a diagnosis of folliculitis.
Which recommendation is most appropriate for treatment of M.M.s folliculitis?
A. Trimethoprim/sulfamethoxazole 1 double-strength tablet orally twice daily for 10 days.
B. Cephalexin 250 mg orally four times daily for 10 days.
C. Warm saline compresses.
D. Ciproflxacin 500 mg orally twice daily for 10 days.
Answer: C
This patient has a simple folliculitis that has not

progressed. Folliculitis is usually a self-limiting


infection that seldom requires antimicrobial treatment,
especially with small lesions. Warm saline compresses
to the infected area can help promote drainage. Although
outbreaks of CA-MRSA can occur, the best approach for
this patient is supportive care; also, watch to make sure
that the infection does not progress.
=====================================
Patient Case
23.K.M. is a 32-year-old woman who presents to the clinic with pain, redness, and swelling below her left
knee
cap. It started about a week ago with what she describes as a bug bite that kept getting more sore, red, and
swollen. She went to the emergency clinic 2 days ago, where she was given a prescription for cephalexin,
but
she states the area keeps getting more painful and red despite the antibiotics. K.M. is an otherwise healthy
woman with no known drug allergies. She works as an elementary teacher and goes to the gym regularly.
Physical examination reveals an erythematous and inflmed area with a necrotic center below the left knee
cap that is very warm to the touch. K.M.s vital signs are normal, and she is afebrile. Incision and drainage
are performed in the offie, and 15 mL of purulent flid is sent for culture and sensitivities. Which is the best
recommendation for empiric coverage of K.M.s cellulitis?
A. Penicillin VK 500 mg orally every 6 hours for 10 days.
B. Vancomycin 1 g intravenously every 12 hours for 10 days.
C. Trimethoprim/sulfamethoxazole 2 double-strength tablets orally twice daily for 10 days.
D. Dicloxacillin 250 mg orally four times daily for 10 days.
. Answer: C
In this case, CA-MRSA should be highly considered.
Community-acquired MRSA is commonly described as
an infection that begins as a bug bite and then develops
into a progressive cellulitis. This patient received
appropriate coverage with cephalexin for common
cellulitis, which would cover the most common and
even the most virulent organisms except for MRSA, and
the infection has continued to progress. The patient is
exposed to environments in which CA-MRSA can be a
risk (elementary school, gym); therefore, coverage for
CA-MRSA is warranted. Penicillin and dicloxacillin
would not be appropriate choices for MRSA. Although
vancomycin would offer appropriate coverage, other, more
convenient, oral options are available. Trimethoprim/
sulfamethoxazole would be the best choice in this case,
but at a dose twice that used for uncomplicated UTIs
=========================================
Patient Case
24. A.T. is a 55-year-man who was treated for his fist recurrent CDI 3 weeks ago with another

treatment course
of metronidazole 500 mg orally three times daily for 10 days. He has not taken antibiotics or any
other
medications in the past 3 weeks, and he now comes to the clinic with severe abdominal pain and
frequent
loose stools for the past few days. The C. diffiile toxin immunoassay comes back positive, and he is
given
a diagnosis of recurrent CDI. Which is the best recommendation for his recurrent infection?
A. Metronidazole 500 mg orally three times daily for 4 weeks.
B. Vancomycin orally tapered over 4 weeks, followed by 4 weeks of pulse dosing.
C. Fidaxomicin 200 mg daily for 10 days.
D. Vancomycin 125 mg orally four times daily for 10 days.
. Answer: B
Because of the potential for neurotoxicity, metronidazole
use is not recommended after the fist recurrence of
CDI. Although fiaxomicin would be a good option
and has decreased recurrent infections compared with
vancomycin, the dose should be 200 mg twice daily.
Vancomycin is the best choice, but because this is the
patients second recurrent infection, a tapered regimen
followed by pulse dosing would be the recommended
dosing, not 125 mg four times daily for 10 days.
Book
25. P.E. is a 56-year-old man who comes to the clinic with
a 3-day history offever, chills, pleuritic chest pain, malaise, and cough productive of sputum. In the clinic,
his
temperature is 102.1F (38.9C) (all other vital signs are
normal). His chest radiograph shows consolidation in
the right lower lobe. His white blood cell count (WBC)
is 14,400/mm3, but all other laboratory values are normal. He is given a diagnosis of community-acquired
pneumonia (CAP). He has not received any antibiotics
in 5 years and has no chronic disease states. Which is
the best empiric therapy for P.E.?
A. Doxycycline 100 mg orally twice daily.
B. Cefuroxime axetil 250 mg orally twice daily.
C. Levofloxacin 750 mg/day orally.
D. Trimethoprim/sulfamethoxazole double
strength orally twice daily.
. Answer: A
The patient has CAP that does not require hospitalization

(CURB-65 score is 1 at most [no mention of mental


status]). Because he has not received any antibiotics
in the past 3 months and has no comorbidities, he is
at low risk of DRSP. Therefore, the drug of choice is
either a macrolide or doxycycline. Cefuroxime is not
recommended for treatment of CAP. Fluoroquinolones
are only recommended if the patient has had recent
antibiotics or has comorbidities. Trimethoprim/
sulfamethoxazole is not used for CAP.
===================================
26. H.W. is a 38-year-old woman who presents with high
temperature, malaise, dry cough, nasal congestion,
and severe headaches. Her symptoms began suddenly 3 days ago, and she has been in bed since then.
She reports no other illness in her family, but several
people have recently called in sick at work. Which is
best for H.W.?
A. Azithromycin 500 mg, followed by 250 mg/day
orally, for 4 more days.
B. Amoxicillin/clavulanic acid 875 mg orally
twice daily.
C. Oseltamivir 75 mg twice daily orally for 5
days.
D. Symptomatic treatment only.
. Answer: D
The symptoms of this patient (high temperature, malaise,
dry cough, nasal congestion, and severe headaches) are
most consistent with influenza; therefore, an antibacterial
agent would not affect recovery. Oseltamivir should be
initiated within 48 hours of symptom onset, so because
this patient is more than 3 days out from symptom onset,
oseltamivir will not affect recovery. Because of the viral
etiology and time since symptom onset, symptomatic
treatment is all that is indicated.
===================================
27. A study is designed to assess the risk of pneumococcal pneumonia in elderly patients 10 years or more
after their pneumococcal vaccination compared
with elderly patients who have never received the
vaccination. Which study design is best?
A. Case series.
B. Case-control study.
C. Prospective cohort study.
D. Randomized clinical trial.
. Answer: B
A case-control study would be the most appropriate study
design because it is the most ethical, cost-effective, timely

methodology. A stronger study designfor instance,


a prospective cohort study or a randomized controlled
trialhas many disadvantages if used to answer this
question. In a prospective cohort study, too many patients
would need to be observed because of the relatively low
incidence of confirmed pneumococcal pneumonia. This
study would therefore be too costly and take too long to
complete. Randomized controlled trials also have many
disadvantages in this situation. First, patients would need
to be vaccinated and then observed for at least 10 years.
Second, too many patients would need to be observed
because of the relatively low incidence of confirmed
pneumococcal pneumonia. Third, it would be unethical
to randomize half of the patients to no vaccination. This
study would therefore be too costly, unethical, and timeconsuming. A case series would evaluate only a few
patients given a diagnosis of pneumococcal pneumonia
10 or more years after vaccination. It would not provide
comparative data, nor would it provide a strong study
design.
======================================
28. S.C. is a 46-year-old woman who presents to the
clinic with purulent nasal discharge, nasal and facial
congestions, headaches, fever, and dental pain. Her
symptoms began about 10 days ago, improved after
about 4 days, and then worsened again a few days
later. Which is the best empiric therapy for S.C.?
A. Cefpodoxime 200 mg twice daily.
B. Clindamycin 300 mg oral four times daily.
C. Amoxicillin/clavulanate 875 mg/125 mg every
12 hours.
D. No antibiotic therapy needed because this is a
typical viral infection
Answer: C
This patient has symptoms suggestive of bacterial
sinusitis, including two major symptoms and a few
minor symptoms. Because the symptoms improved
and then worsened suggests a bacterial sinusitis that
followed a viral infection. Although the combination of
cefpodoxime and clindamycin is an option for sinusitis
in penicillin-allergic patients, it is not recommended to
give either of these alone for treatment. The best option
is amoxicillin/clavulanate, which has activity against
organisms commonly seen in bacterial sinusitis and is
considered a first-line agent.
============================================

29. N.R. is a 28-year-old woman who presents to the


clinic with a 2-day history of dysuria, frequency,
and urgency. She has no significant medical history,
and the only drug she takes is oral contraceptives.
Which is the best empiric therapy for N.R.?
A. Oral nitrofurantoin extended release (ER) 100
mg twice daily for 3 days.
B. Oral ciprofloxacin 500 mg twice daily for 7
days.
C. Oral trimethoprim/sulfamethoxazole double
strength twice daily for 3 days.
D. Oral cephalexin 500 mg four times daily for 3
days.
. Answer: C
Although nitrofurantoin is a recommended first-line
agent, the therapy duration is too short for its use.
Because this patient has no contraindications to the use
of trimethoprim/sulfamethoxazole or nitrofurantoin,
and trimethoprim/sulfamethoxazole resistance rates are
not mentioned as being high, fluoroquinolones would
not be considered appropriate as first-line therapy in
this particular case. In addition, 7 days of therapy
is not necessary. The best choice for this patient is
trimethoprim/sulfamethoxazole double strength twice
daily orally for 3 days. The patient should be counseled
about the potential interaction between antibiotics and
oral contraceptives. -Lactams are not as effective as
trimethoprim/sulfamethoxazole, and data are limited on
their use for 3 days.
==================================
30. B.Y. is an 85-year-old woman who is bedridden and
lives in a nursing home. She is chronically catheterized, and her urinary catheter was last changed 3
weeks ago. Today, her urine is cloudy, and a urinalysis shows many bacteria. B.Y. is not noticing any
symptoms. A urine culture is obtained. Which option is best for B.Y.?
A. No therapy because she is chronically catheterized and has no symptoms.
B. No antibiotic therapy, but the catheter should
be changed.
C. Oral ciprofloxacin 500 mg twice daily for 7
days and a new catheter.
D. Oral ciprofloxacin 500 mg twice daily for
1421 days without a change in catheter.
. Answer: A
For the asymptomatic patient who is bedridden and
chronically catheterized, with cloudy urine and bacteria
shown by urinalysis, no therapy is indicated. All patients
with chronic urinary catheters will be bacteriuric.

Because this patient is asymptomatic, the catheter does


not need to be replaced. Ifshe were symptomatic, catheter
replacement might be indicated. Antibiotics are not
indicated; however, a 7-day course would be appropriate
if treatment were instituted. A long course of treatment
only increases the risk of acquiring resistant organisms.
=====================================
31. V.E. is a 44-year-old man who presents to the emergency department with a warm, erythematous, and
painful right lower extremity. There is no raised border at the edge of the infection. Three days ago, he
scratched his leg on a barbed wire fence on his property. His temperature has been as high as 101.8
(38 ) with chills. Doppler studies of his lower extremity are negative. Blood cultures were drawn,
and they are negative to date. Which is the best empiric therapy for V.E.?
A. Nafcillin 2 g intravenously every 6 hours. The
infection may worsen, and necrotizing fasciitis
needs to be ruled out.
B. Penicillin G, 2 million units intravenously
every 4 hours. This is probably erysipelas.
C. Piperacillin/tazobactam 3.375 g intravenously
every 6 hours. Surgical debridement is vitally
important.
D. Enoxaparin 80 mg subcutaneously twice daily
and warfarin 5 mg/day orally.
Answer: A
Because cellulitis (which the patient appears to have)
is usually caused by Streptococcus or Staphylococcus,
nafcillin is the drug of choice (vancomycin could be
initiated empirically if MRSA were a concern in this
patient). Necrotizing fasciitis needs to be ruled out because
other organisms may be involved, and surgery would be
crucial. Although penicillin is the treatment of choice
for erysipelas, the patient probably has acute cellulitis
(there is no raised border at the edge of the infection,
which is indicative of erysipelas). Although piperacillin/
tazobactam has activity against both Streptococcus and
Staphylococcus, this treatment is too broad spectrum for
an acute cellulitis. Because Doppler studies are negative,
the likelihood ofa deep venous thrombosis is low.
===========================================
32. R.K. is a 36-year-old woman who presents to the
emergency department with a severe headache and
neck stiffness. Her temperature is 99.5 (37.5 ).
After a negative computed tomographic scan of the
head, a lumbar puncture is performed, showing the
following: glucose 54 mg/dL (peripheral, 104), protein 88 mg/dL, and WBC 220/mm3 (100% lymphocytes).
The Gram stain shows no organisms. Which

option describes the best therapy for R.K.?


A. This is aseptic (probably viral) meningitis, and
no antibiotics are necessary.
B. Administer ceftriaxone 2 g intravenously every
12 hours until the cerebrospinal fluid (CSF)
cultures are negative for bacteria.
C. Administer ceftriaxone 2 g intravenously every
12 hours and vancomycin 1000 mg intravenously every 12 hours until the CSF cultures
are negative for bacteria.
D. Administer acyclovir 500 mg intravenously
every 8 hours until the CSF culture results are
Complete.
Answer: C
Even if a patient is believed to have aseptic meningitis
after analysis of the CSF, antibiotics need to be given
until CSF cultures are negative. In empiric therapy
for bacterial meningitis in adults (i.e., when the CSF
Gram stain is negative), ceftriaxone should be used
in combination with vancomycin. The vancomycin is
required for activity against resistant S. pneumoniae.
Although the symptoms and CSF results are similar to
what is expected for herpes simplex encephalitis, the
use of acyclovir alone in this patient is inappropriate.
Antibacterials must be used as well. Viral meningitis
is generally caused by coxsackie virus, echovirus, and
enterovirus, which are not treated with acyclovir.
==========================================
24. L.G. is a 49-year-old woman with a history of mitral
valve prolapse. She presents to her physicians office
with malaise and a low-grade fever. Her physician
notes that her murmur is louder than normal and orders blood cultures and an echocardiogram. A large
vegetation is observed on L.G.s mitral valve, and
her blood cultures are growing Enterococcus faecalis (susceptible to all antibiotics). Which is the best
therapy for L.G.?
A. Penicillin G plus gentamicin for 2 weeks.
B. Vancomycin plus gentamicin for 2 weeks.
C. Ampicillin plus gentamicin for 46 weeks.
D. Cefazolin plus gentamicin for 46 weeks.
Answer: C
Enterococcal endocarditis should be treated for 46
weeks. The 2-week treatment regimen is indicated only
for streptococcal endocarditis. There is also no indication
that the patient is penicillin allergic; thus, vancomycin
should not be used as first-line treatment. Ampicillin
plus gentamicin for 46 weeks is the regimen of choice

for penicillin-susceptible enterococcal endocarditis.


Cephalosporins have no activity against Enterococcus;
therefore, the regimen with cefazolin is inappropriate.
=======================================
25. N.L. is a 28-year-old woman with no significant
medical history. She reports to the emergency department with fever and severe right lower quadrant
pain. She has had a dull pain for the past few days,
but it suddenly became severe during the past 8
hours. Her temperature is 103.5 (39.7 ), and she
has rebound tenderness on abdominal examination.
She is taken to surgery immediately, where a perforated appendix is diagnosed and repaired. Which is
the best follow-up antibiotic regimen?
A. Vancomycin 1000 mg intravenously every 12
hours plus metronidazole 500 mg intravenously every 8 hours.
B. Ceftriaxone 1 g/day intravenously plus ciprofloxacin 400 mg intravenously every 12 hours.
C. Ertapenem 1 g/day intravenously.
D. No antibiotics needed after surgical repair of a
perforated appendix.
. Answer: C
A perforated appendix requires antibiotics after surgery
for an intra-abdominal infection. The combination of
vancomycin and metronidazole does not have adequate
activity against aerobic, gram-negative organisms
(e.g., E. coli). The combination of ceftriaxone and
ciprofloxacin does not have adequate activity against
anaerobic organisms (e.g., B. fragilis group). Ertapenem
is a good choice for intra-abdominal infections, although
it has limited activity against Enterococcus.
=================================================
Patient Case
26. R.L. is a 68-year-old man who presents to the emergency department with coughing and shortness
ofbreath.
His symptoms, which began 4 days ago, have worsened during the past 24 hours. He is coughing up
yellowgreen sputum, and he has chills with a temperature of 102.4 (39 ). His medical history includes
coronary artery disease with a myocardial infarction 5 years ago, congestive heart failure, hypertension, and
osteoarthritis. He rarely drinks alcohol and has not smoked since his myocardial infarction. His medications
on admission include lisinopril 10 mg/day, hydrochlorothiazide 25 mg/day, and acetaminophen 650 mg four
times/day. On physical examination, he is alert and oriented, with the following vital signs: temperature
101.8 (38 ), heart rate 100 beats/minute, respiratory rate 24 breaths/minute, and blood pressure 142/94
mm Hg. His laboratory results are normal except for blood urea nitrogen (BUN) 32 mg/dL (serum creatinine
1.23 mg/dL). Blood gases are pH 7.44; Pco 35; Po 82; and O sat 90%. A sputum specimen is not
available. If R.L. were hospitalized, which would be the best empiric therapy for him?
A. Ampicillin/sulbactam 1.5 g intravenously every 6 hours.
B. Piperacillin/tazobactam 4.5 g intravenously every 6 hours plus gentamicin 180 mg intravenously
every 12 hours.

C. Ceftriaxone 1 g intravenously every 24 hours plus azithromycin 500 mg/day intravenously.


D. Doxycycline 100 mg intravenously every 12 hours.
. Answer: C
Although ampicillin/sulbactam has good activity against
H. influenzae, Moraxella catarrhalis, and S. pneumoniae
(but not drug-resistant S. pneumoniae [DRSP]), it has no
activity against atypical organisms (L. pneumophila,
Mycoplasma pneumoniae, Chlamydia pneumoniae).
Current recommendations are to include a macrolide
with a -lactam antibiotic for hospitalized patients with
CAP. Piperacillin/tazobactam has good activity against
H. influenzae, M. catarrhalis, and S. pneumoniae (but not
DRSP) and, with gentamicin, is excellent for pneumonia
caused by most gram-negative organisms. However,
this increased activity is not necessary for CAP, and the
combination has no activity against atypical organisms.
Ceftriaxone plus azithromycin is the best initial choice.
It has excellent activity against atypical organisms
(because of azithromycin), H. influenzae, M. catarrhalis,
and S. pneumoniae (even intermediate DRSP). Although
doxycycline has activity against atypical organisms and
most of the typical organisms that cause CAP, it is not
recommended as monotherapy in hospitalized patients.
In addition, its activity against S. pneumoniae may be
limited (if the patient lives in an area with extensive
DRSP). Doxycycline would not be the best initial choice.
=========================================
Patient Case
27. B.P. is a 66-year-old woman who underwent a two-vessel coronary artery bypass graft 8 days ago and
has
been on a ventilator in the surgical intensive care unit since then. Her temperature is now rising, and a
tracheal aspirate shows many WBCs and gram-negative rods. Her medical history includes coronary artery
disease with a myocardial infarction 2 years ago, COPD, and hypertension. Which is the best empiric
therapy for B.P.?
A. Ceftriaxone 1 g/day intravenously plus gentamicin 480 mg intravenously every 24 hours plus linezolid
600 mg intravenously every 12 hours.
B. Piperacillin/tazobactam 4.5 g intravenously every 6 hours.
C. Levofloxacin 750 mg/day intravenously plus linezolid 600 mg intravenously every 12 hours.
D. Cefepime 2 g intravenously every 12 hours plus tobramycin 480 mg intravenously every 24 hours plus
vancomycin 15 mg/kg intravenously every 12 hours.
. Answer: D
Ceftriaxone plus gentamicin plus linezolid is not good
empiric therapy because ceftriaxone has limited activity
against P. aeruginosa, and gentamicin has variable
activity against P. aeruginosa, depending on the
institution. Because the patient has been on a ventilator

and in an intensive care unit for 8 days, she is at increased


risk of nosocomial pneumonia, specifically caused by
P. aeruginosa (and possibly MRSA, depending on the
institution). Although piperacillin/tazobactam has good
activity against most common causes of nosocomial
pneumonia (including P. aeruginosa), the most recent
guidelines recommend two antibiotics with activity
against P. aeruginosa for patients with severe nosocomial
pneumonia, and she may require an antibiotic with
MRSA activity. Levofloxacin has only moderate activity
against P. aeruginosa, and two drugs should be used.
Cefepime plus tobramycin plus vancomycin is the best
empiric therapy because it includes two antibiotics with
excellent activity against P. aeruginosa and another
agent for MRSA.
==================================
Patient Case
28. B.P., who eventually improves, is transferred to a regular floor. She cannot remember receiving any
recent
vaccinations. Which is the best vaccination recommendation for this patient?
A. B.P. needs no vaccinations.
B. B.P. should receive the pneumococcal vaccine now and the influenza vaccine in the fall.
C. B.P. should receive the influenza vaccine in the fall, but because of her current infection, the
pneumococcal vaccine is unnecessary.
D. B.P. should receive the pneumococcal vaccine now, but she is not in a group in which the influenza
vaccine is recommended.
. Answer: B
This patient should receive vaccinations now. There are
no contraindications to receiving either pneumococcal
or influenza vaccine immediately after an episode of
pneumonia. It is best to vaccinate whenever patients
are available. This patients age and medical history put
her at risk of both pneumococcal disease and influenza.
Therefore, administration of pneumococcal and
influenza vaccines is indicated (if it is during the middle
ofinfluenza season and she was not vaccinated in the fall,
she can receive the influenza vaccine now). The patients
age places her in a group needing the pneumococcal
vaccine, and everyone should receive the influenza
vaccine. The causative agent for her current infection
does not affect the recommendation for vaccination.
===========================================
Patient Case
29. G.N. is a 62-year-old woman who presents to the clinic with a 3-day history of urinary frequency and
dysuria. During the past 24 hours, she has had nausea, vomiting, and flank pain. G.N. has a history of type 2

diabetes mellitus, which is poorly controlled with some diabetic-related complications. G.N. also has
hypertension and a history of several episodes of deep venous thrombosis. Her medications include
glyburide 5
mg/day orally, enalapril 10 mg orally twice daily, warfarin 3 mg/day orally, and metoclopramide 10 mg four
times/day. On physical examination, she is alert and oriented, with the following vital signs: temperature
102.8 (39 ); heart rate 120 beats/minute; respiratory rate 16 breaths/minute; blood pressure (supine):
140/75 mm Hg; and blood pressure (standing) 110/60 mm Hg. Her laboratory values are within normal
limits except for increased international normalized ratio 2.7; BUN 26 mg/dL; serum creatinine 1.88 mg/
dL; and WBC 12,000 (78 polymorphonuclear leukocytes, 7 band neutrophils, 10 lymphocytes, and 5
monocytes). Her urinalysis shows turbidity, 2+ glucose; pH 7.0; protein 100 mg/dL; 50100 WBC; + nitrites;
35
red blood cells; and many bacteria and + casts. Which is the best empiric therapy for G.N.?
A. Trimethoprim/sulfamethoxazole double strength orally twice dailyduration of antibiotics: 7 days.
B. Ciprofloxacin 400 mg intravenously twice daily and then 500 mg orally twice dailyduration of
antibiotics: 10 days.
C. Gentamicin 140 mg intravenously every 24 hoursduration of antibiotics: 3 days.
D. Tigecycline 100 mg once; then 50 mg every 12 hours and then doxycycline 100 mg twice dailyduration
of antibiotics: 10 days.
Answer: B
Although the treatment duration is correct for this
patients diagnosis (7 days), oral trimethoprim/
sulfamethoxazole is inappropriate for complicated
pyelonephritis. It will also interact with warfarin,
increasing the risk of bleeding. Ciprofloxacin 400 mg
intravenously twice daily and then 500 mg orally twice
daily for 10 days is an appropriate choice and duration
(714 days) for this complicated pyelonephritis (it may
also interact with warfarin, but to a lesser extent than
trimethoprim/sulfamethoxazole). It would be expected
to have activity against the common organisms causing
complicated pyelonephritis. Gentamicin for 3 days is
too short a treatment duration, and tigecycline, followed
by doxycycline, is not recommended for complicated
pyelonephritis (although tigecycline is found unchanged
in the urine).
======================================
Patient Case
30. G.N. returns to the clinic in 6 months with no urinary symptoms, but her chief concern is now an ulcer
on her right foot. She recently returned from a vacation in Florida and thinks she might have stepped on
something while walking barefoot on the beach. Her foot is not sore but is red and swollen around the ulcer.
The ulcer is deep, and the infection may involve the underlying bone. Her medications are the same as
before. Vital signs are stable, and there is nothing significant on physical examination except for the right
foot
ulcer. Laboratory values are within normal limits (serum creatinine 0.86 mg/dL). Which best describes the

organism(s) likely responsible for G.N.s foot ulcer?


A. Multiple anaerobic organisms.
B. P. aeruginosa.
C. S. aureus.
D. Polymicrobial with gram-positive, gram-negative, and anaerobic organisms.
Answer: D
Diabetic foot infections are generally polymicrobial
(average organisms, 2.55.8).
========================================
Patient Case
31. Which is the best empiric therapy for G.N.?
A. Nafcillin 2 g intravenously every 6 hoursduration of antibiotics: 612 weeks.
B. Tobramycin 120 mg intravenously every 12 hours plus levofloxacin 750 mg/day intravenouslyduration
of antibiotics: 12 weeks.
C. Ampicillin/sulbactam 3 g intravenously every 6 hoursduration of antibiotics: 23 weeks.
D. Below-the-knee amputation followed by ceftriaxone 1 g intravenously every 24 hoursduration of
antibiotics: 1 week.
. Answer: C
Nafcillin has excellent activity against gram-positive
organisms, but it would miss the gram-negative organisms
and anaerobes often involved in moderate to severe
diabetic foot infections. Tobramycin and levofloxacin
would be good against aerobic organisms, but levofloxacin
has only limited activity against anaerobes. Tobramycin
may also not be a good choice for a patient with diabetes
mellitus with long-term complications (because of the
increased risk of nephrotoxicity). -Lactamase inhibitor
combinations are good agents because they have activity
against the organisms that are often involved. At this
time, a regimen active against P. aeruginosa is probably
not necessary. Treatment duration may need to be
extended if the bone is involved. Aggressive antibiotic
treatment often prevents the need for an amputation.
=====================================
Patient Case
32. W.A. is a 55-year-old man who presents with weight loss, malaise, and severe back pain and spasms that
have progressed during the past 2 months. He has also experienced loss of sensation in his lower extremities.
Four months before this admission, he had surgery for a fractured tibia, followed by an infection treated
with unknown antibiotics. W.A. has hypertension and diverticulitis. On physical examination, he is alert
and oriented, with the following vital signs: temperature 99.4 (37.4 ); heart rate 88 beats/minute;
respiratory rate 14 breaths/minute; and blood pressure 130/85 mm Hg. His laboratory values are within
normal
limits, except for WBC 14,300, erythrocyte sedimentation rate 89 mm/hour, and C-reactive protein 12 mg/
dL. Magnetic resonance imaging shows bony destruction of lumbar vertebrae 1 and 2, which is confirmed
by a bone scan. A computed tomographyguided bone biopsy shows gram-positive cocci in clusters. Which

is the best initial therapy for W.A.?


A. Vancomycin 15 mg/kg intravenously every 12 hoursduration of antibiotics: 6 weeks.
B. Nafcillin 2 g intravenously every 6 hoursduration of antibiotics: 2 weeks.
C. Levofloxacin 750 mg/day orallyduration of antibiotics: 6 weeks.
D. Ampicillin/sulbactam 3 g intravenously every 6 hoursduration of antibiotics: 2 weeks.
Answer: A
Because sensitivities of the gram-positive organism are
still unknown, vancomycin is the best choice. In addition,
the therapy duration for osteomyelitis is 46 weeks.
Therefore, the 2-week duration with nafcillin is too short.
Although levofloxacin is advantageous because it can be
given orally, it will probably not achieve adequate bone
concentrations to eradicate S. aureus (the most likely
organism). Ampicillin/sulbactam is effective against S.
aureus (except for MRSA); its broad spectrum of activity
is not necessary in this situation, and the duration is too
short.
======================================
Patient Case
33. D.M. is a 21-year-old university student who presents to the emergency department with the worst
headache
of his life. During the past few days, he has felt slightly ill but has been able to go to class regularly and eat
and drink adequately. This morning, he awoke with a terrible headache and pain whenever he moved his
neck. He has no significant medical history and takes no medications. He cannot remember the last time
he received a vaccination. On physical examination, he is in extreme pain (10/10) with the following vital
signs: temperature 102.4 (39.1 ); heart rate 110 beats/minute; respiratory rate 18 breaths/minute; and
blood pressure 130/75 mm Hg. His laboratory values are within normal limits, except for WBC 22,500 (82
polymorphonuclear leukocytes, 11 band neutrophils, 5 lymphocytes, and 2 monocytes). A computed
tomography scan ofthe head is normal, so a lumbar puncture is performed with the following results: glucose
44 mg/dL (peripheral, 110); protein 220 mg/dL; and WBC 800/mm3 (85% neutrophils, 15% lymphocytes).
Gram staining shows abundant gram-negative cocci. Which is the best empiric therapy for D.M.?
A. Penicillin G 4 million units intravenously every 4 hours plus dexamethasone 4 mg intravenously
every 6 hours.
B. Ceftriaxone 2 g intravenously every 12 hours.
C. Ceftriaxone 2 g intravenously every 12 hours plus dexamethasone 4 mg intravenously every 6 hours.
D. Ceftriaxone 2 g intravenously every 12 hours plus vancomycin 1000 mg intravenously every 12 hours.
Answer: B
From his presentation and laboratory values, this
patient has bacterial meningitis. The gram-negative
cocci on Gram stain are most likely N. meningitidis.
Penicillin is effective against N. meningitidis; however,
some strains are resistant, and until culture results are
received, it is unwise to use this agent alone. Ceftriaxone
alone is effective for meningococcal meningitis, and
this is the best Answer: (although some may continue
to use vancomycin until the cultures actually grow N.

meningitidis). Dexamethasone is beneficial only in adults


with pneumococcal meningitis (not meningococcal
meningitis). Ceftriaxone is the appropriate empiric
antibiotic therapy in this situation. Vancomycin is
generally used empirically because of its activity against
highly penicillin-resistant S. pneumoniae. Because this
is probably not pneumococcal meningitis, vancomycin
can be discontinued.
=======================================
Patient Case
34. After D.M.s diagnosis, there is concern about prophylaxis. Which is the best recommendation for
meningitis prophylaxis?
A. The health care providers in close contact with D.M. should receive rifampin 600 mg every 12 hours
for four doses.
B. Everyone in D.M.s dormitory and in all ofhis classes should receive rifampin 600 mg/day for 4 days.
C. Everyone in the emergency department at the time of D.M.s presentation should receive the
meningococcal conjugate vaccine.
D. Everyone in the emergency department at the time of D.M.s presentation should receive rifampin 600
mg every 12 hours for four doses.
Answer: A
Only people in close contact to a patient with
meningococcal meningitis require prophylaxis (primarily
those who live closely with the patient and those who
are exposed to oral secretions). The correct regimen is
rifampin 600 mg every 12 hours for four doses. Although
the vaccine is a good idea for those at future risk of
acquiring this infection (e.g., college students living in
dormitories), its use during an outbreak is very limited.
=======================================
Patient Case
35. T.S. is a 48-year-old man who presents to the emergency department with fever, chills, nausea/vomiting,
anorexia, lymphangitis in his right hand, and lower back pain. He has no significant medical history except
for kidney stones 4 years ago. He has no known drug allergies. He is homeless and was an intravenous drug
abuser (heroin) for the past year but quit 2 weeks ago. On physical examination, he is alert and oriented,
with the following vital signs: temperature 100.8 (38 ); heart rate 114 beats/minute; respiratory rate 12
breaths/minute; and blood pressure 127/78 mm Hg. He has a faint systolic ejection murmur, and his right
hand is erythematous and swollen. His laboratory values are all within normal limits. He had an HIV test
1 year ago, which was negative. One blood culture was obtained that later grew MSSA. Two more cultures
were obtained that are now growing gram-positive cocci in clusters. A transesophageal echocardiogram
shows vegetation on the mitral valve. Which is the best therapeutic regimen for T.S.?
A. Nafcillin intravenous therapyantibiotic duration: 2 weeks.
B. Nafcillin intravenously plus rifampin therapyantibiotic duration: 6 weeks or longer.
C. Nafcillin intravenously plus gentamicin intravenous therapyantibiotic duration: 2 weeks of both
antibiotics.
D. Nafcillin intravenously plus gentamicinantibiotic duration: 6 weeks (nafcillin) with gentamicin for

the first 35 days.


. Answer: D
The treatmentduration is too short(nafcillin intravenously
?2 weeks) for S. aureus endocarditis. Only streptococcal
endocarditis can be treated for 2 weeks. Although
nafcillin intravenously plus rifampin therapy for 6
weeks or longer is an appropriate duration for MSSA,
the rifampin does not need to be added in patients with
native valve endocarditis. Nafcillin intravenously plus
gentamicin intravenously ?2 weeks is too short for S.
aureus endocarditis. Nafcillin intravenously ?6 weeks
with gentamicin for the first 35 days is the recommended
treatment for MSSA endocarditis. Gentamicin needs to
be added for only 35 days to decrease the duration of
bacteremia.
====================================
Patient Case
36. Six months after treatment ofhis endocarditis, T.S. is visiting his dentist for a tooth extraction.
Which antibiotic is best for prophylaxis?
A. Tooth extractions do not require endocarditis prophylaxis.
B. Administer amoxicillin 2 g 1 hour before the extraction.
C. Administer amoxicillin 3 g 1 hour before the extraction and 1.5 g 6 hours for four doses after the
extraction.
D. T.S. is not at increased risk of endocarditis and does not need prophylactic antibiotics.
Answer: B
This patient is at increased risk of endocarditis because
of his history of the disease. Tooth extractions require
prophylaxis for those at risk. Amoxicillin 2 g, 1 hour
before the tooth extraction, is the current recommended
dose. The 2-g dose is adequate for protection, and a
follow-up dose is not needed. Amoxicillin 3 g, 1 hour
before the extraction, and 1.5 g, 6 hours for four doses
after the extraction, is the older recommended dose. A
follow-up dose is not needed.
===========================================
Patient Case
37. You are a pharmacist who works closely with the surgery department to optimize therapy for patients
undergoing surgical procedures at your institution. The surgeons provide you with principles of surgical
prophylaxis that they believe are appropriate. Which is the best practice for optimizing surgical prophylaxis?
A. Antibiotics should be re-dosed for extended surgical procedures; re-dose if the surgery lasts longer
than 4 hours or involves considerable blood loss.

B. All patients should be given antibiotics for 24 hours after the procedure; this will optimize prophylaxis.
C. Preoperative antibiotics can be given up to 4 hours before the incision; this will make giving the
antibiotics logistically easier.
D. Vancomycin should be the antibiotic of choice for surgical wound prophylaxis because of its long
half-life and activity against MRSA
. Answer: A
Re-dosing antibiotics for surgical prophylaxis is very
importantespecially for antibiotics with short halflives, for extended surgical procedures, or for when
there is extensive blood loss. Antibiotics given beyond
the surgical procedure are generally unnecessary and
only increase the potential for adverse drug reactions
and resistant bacteria. Although preoperative antibiotics
given up to 4 hours before the incision may improve the
logistics of administering surgical prophylaxis, study
results show that antibiotics need to be given as close
to the time of the incision as possible (definitely within
2 hours). Vancomycin should not be used routinely for
surgical prophylaxis. The Centers for Disease Control and
Prevention does not recommend the use of vancomycin
for routine surgical prophylaxis other than in a patient
with life-threatening allergy to -lactam antibiotics.
38. A 78-year-old man is admitted to the general medicine floor with flank pain, altered mental
status, and fever. Urinalysis is significant for white blood cell (WBC) count of 20 cells/high-power
field (hpf), large leukocyte esterase, and nitrite positive. Initial urine Gram stain (clean catch sample)
reveals gram-negative rods. Assuming institutional antibiogram data would not influence selection,
what is the best initial therapy?
A. Nitrofurantoin 100 mg orally twice daily
B. Ciprofloxacin 400 mg intravenously twice daily
C. Ampicillin/sulbactam 1.5 g intravenously every 6 hours
D. Piperacillin/tazobactam 3.375 g intravenously every 6 hours
Answer b
=============================================
39-A 77-year-old woman is admitted to the medical step-down unit with severe dehydration and
810 watery stools per day. Three weeks ago, she completed a 10-day course of metronidazole for
Clostridium difficile infection. Current laboratory values include a white blood cell (WBC) count of
19,000 cells/mm3, serum creatinine 1.2 mg/dL (baseline 1.0 mg/dL), and blood urea nitrogen (BUN)
29 mg/dL. Patient is tachycardic (107 beats/minute) with all other vital signs stable. The patient
reports no known drug allergies. C. difficile toxin B polymerase chain reaction (PCR) is positive.
Other concurrent medications include omeprazole 20 mg/day, atorvastatin 10 mg/day, lisinopril 10

mg/day, and hydrochlorothiazide 25 mg/day. What is the best initial management decision at this
point?
A. Initiate metronidazole 500 mg orally every 8 hours.
B. Initiate vancomycin 125 mg orally every 6 hours.
C. Initiate metronidazole 500 mg intravenously every 8 hours and vancomycin 125 mg orally
every 6 hours.
D. Initiate metronidazole 500 mg orally every 8 hours and fidaxomicin 200 mg every 12 hours.
Answer Explanation
B. Initiate vancomycin 125 mg orally every 6 hours.
Vancomycin therapy would be most appropriate, given the clinical status (including elevated white blood
cell [WBC] count), recurrence, and high-risk patient (elderly and receiving proton pump inhibitor [PPI]
therapy). The patient is hemodynamically stable at this point; thus, combination therapy would not be
indicated. Fidaxomicin therapy has not been studied in combination, although it could perhaps be an option
in this patient as monotherapy.
============================================
40Vignette:
A 55-year-old woman with a medical history significant for a
bioprosthetic mitral valve replacement, hypertension, and depression has an
appointment with the dentist next week to have her teeth cleaned. Her current
medications include aspirin 81 mg orally daily, candesartan 16 mg orally daily,
hydrochlorothiazide 25 mg orally daily, and escitalopram 10 mg orally daily.
She states that her throat and lips swell when she takes penicillin.
Which antibiotic regimen, if any, would be most appropriate to prevent infective
endocarditis in this patient?
A
Amoxicillin 2 g orally 3060 minutes before the procedure
B
Azithromycin 500 mg orally 3060 minutes before the procedure
Clindamycin 600 mg orally 3060 minutes before and after the
C
procedure
D
No antibiotic prophylaxis required for this patient
Answer Explanation:
This patient requires antibiotic prophylaxis for infective endocarditis because
she has a bioprosthetic mitral valve and is undergoing a dental procedure
(teeth cleaning) that involves manipulation of her gingival tissue. Therefore,
Answer D is incorrect. Only a single dose of antibiotic therapy is required as
prophylactic therapy, with the dose given 3060 minutes before the procedure.
Therefore, Answer C is incorrect. This patient appears to have experienced an
anaphylactic reaction to penicillin in the past; therefore, penicillin and
penicillin-like drugs (such as amoxicillin) should be avoided in this patient
(Answer A). In patients with an allergy to penicillin, a single dose of
clindamycin, azithromycin, or clarithromycin given 3060 minutes before the

procedure is recommended (Answer B).


===================================
41Vignette:
A 37-year-old woman is admitted to the trauma intensive care unit for a closed,
fracture-dislocation of the thoracic spine secondary to a motor vehicle collision.
She is immediately taken to the operating room for stabilization of her spine.
Which statement is true regarding perioperative antibiotic administration in this
patient?
Antibiotics should be administered within 2 hours of the first
A
surgical incision.
B
Antibiotics should be discontinued within 48 hours after surgery.
Intraoperative re-dosing of antibiotics is not required unless surgery
C
lasts more than 8 hours.
Antibiotic prophylaxis is not indicated before surgery for a closed
D
spinal cord injury.
Answer Explanation:
Most antibiotics should be administered within 1 hour of the first surgical
incision. Exceptions are antibiotics that require longer infusion times such as
fluoroquinolones and vancomycin; only in these scenarios would it be
appropriate to administer antibiotics within 2 hours of the incision (Answer A is
incorrect). Intraoperative re-dosing of antibiotics depends on the half-life of the
drug. Re-dosing in the operating room is recommended at an interval of about
2 times the half-life of the agent (Answer C is incorrect). Although open
fractures are associated with more surgical-site contamination, antibiotic
prophylaxis is still indicated for surgical intervention for closed fractures of the
spinal cord (Answer D is incorrect). The duration for most perioperative
antibiotic dosing should be less than 24 hours; however, performance
measures set forth by the Joint Commission specify that antibiotics should be
discontinued within 48 hours after the end of surgery (Answer B is correct).
=======================================
42Vignette:
A 28-year-old woman, who is 8 weeks post-delivery of a healthy baby girl,
presents to her family medicine provider with chills, fevers, and a lump in her
right breast with redness and tightness in the area. She is currently breast
feeding her baby girl.
Medical history: diabetes, cesarean section x 1

Medications: prenantal vitamin, insulin glargine and insulin aspart


Medication allergies: penicillin (rash)
Her physician suspects mastitis and asks for your advice on an appropriate
antibiotic recommendation. Which would be the best recommendation?
A
Dicloxacillin 500 mg orally four times daily
B
Cephalexin 500 mg orally four times daily
C
Clindamycin 300 mg orally four times daily
D
Linezolid 600 mg orally twice daily
Answer Explanation:
Most cases of mastitis are caused by Staphylococcus aureus and therefore,
dicloxacillin, cephalexin, clindamycin or linezolid are likely to provide
coverage.
In the absence of risk factors for methicillin-resistant S.
aureus (MRSA), dicloxacillin or cephalexin would be appropriate
choices. Given her penicillin allergy, dicloxacillin should be avoided due to
risk of cross-reaction. Cephalexin may be an acceptable alternative and may
be tolerated because the patient just has a rash to penicillin, and there are no
reports of hives, desquamation, or respiratory difficulties. However,
clindamycin may be a better alternative for this patient because it will
cover MRSA (methicillin-resistant Staphylococcus aureus), which she may be
at an increased risk of, given her history of diabetes. Linezolid would be higher
cost and is likely not warranted in this situation.
==========================
43Vignette:
A 19-year-old female presents to her family medicine physician with headache,
stiff neck, and fevers that began about 2 days ago.
Vital signs: Temperature 39.4C, respiratory rate 18 breaths/minute, heart rate
97 beats/minute, blood pressure 106/69 mm Hg, pain 7/10
Medical history: non-significant
Medications: ethinyl estradiol/norelgestromin transdermal patch applied once
weekly
Allergies: No known drug allergies
Social history: College student residing in a co-educational dormitory and has
a roommate. Denies tobacco or illicit substance use but does report occasional
alcohol use. Most recently, she reports being at a fraternity party the previous
weekend where she reports ingesting innumerable amounts of beers and
mixed drinks. Sexually active with one partner. She reports that a friend of

hers, who was also at the party and lives in her dormitory, had similar
symptoms.
She is sent to the emergency department for a lumbar puncture and CT scan
which reveals the following:
Head CT: Meningeal inflammation
Lumbar puncture: Normal opening pressure, glucose 45 mg/dL, protein 300
mg/dL, white blood cell count 150 cells/microliter
Microbiology: Blood Gram stain negative; blood culture pending;
cerebrospinal fluid (CSF) Gram stain Gram stainnegative diplococci; CSF
culture pending
Which would be the most appropriate empiric therapy for the patient?
A
Penicillin G 4 million units intravenously every 4 hours
B
Ceftriaxone 2 g intravenously every 12 hours
Ceftriaxone 2 g intravenously every 12 hours and vancomycin 15
C
mg/kg intravenously every 8 hours
Ceftriaxone 2 g intravenouly every 12 hours, vancomycin 15 mg/kg
D
intravenously every 8 hours, and ampicillin 2 g intravenously every
4 hours
Answer Explanation:
Given her risk factors and age, the patient likely has meningitis caused by N.
meningitidis. However, until culture is finalized and susceptibilities return, we
treat empirically for all potential causative pathogens, which are N.
meningitidis and Streptococcus pneumoniae. Penicillin is the drug of choice for
susceptible N. meningitidis, but empiric treatment, according to the guidelines,
should be ceftriaxone and vancomycin. Ampicillin would not be needed unless
she was immunocompromised, asplenic, or older than 50 years.
44Vignette:
A 47-year-old male is admitted to the general medicine floor of your hospital
with a 3-day history of a productive cough with greenish yellow sputum and
shortness of breath. He has a white blood cell count of 13 K/mm3(baseline 5),
serum creatinine of 1.2 mg/dL (baseline 11.3 mg/dL), and Tmax of 38.3C,
and his admission chest X-ray shows a left lower lobe pneumonia. He has
no known drug allergies but a medical history significant for a living related
kidney transplant 10 years ago for end-stage renal disease caused by
polycystic kidney disease. In his history from a transplant clinic note 5 months

ago, you see that he has had no recent treated rejections; is maintained on
tacrolimus 1 mg orally twice daily with goal serum troughs between 2 and 4
ng/mL and prednisone 5 mg daily; and is seen yearly by the transplant
nephrologists but that he has no other history of hospital
or emergency department contact. The medical resident calls you for help with
antibiotic initiation.
Which of the following is the best choice for empiric antibiotic therapy?
Piperacillin/tazobactam 4.5 g intravenously every 6 hours and
A
vancomycin dosed for trough 1520 mcg/mL
Azithromycin 500 mg orally x 1; then 250 mg orally daily x 4 more
B
days
Cefepime 2 g intravenously every 8 hours, tobramycin 7 mg/kg
C
intravenously every 24 hours, and vancomycin dosed for a trough
1520 mcg/dL
Ceftriaxone 1 g intravenously every 24 hours and azithromycin 500
D
mg orally daily
Answer Explanation:
Answer: D.
Although the patient is immunocompromised on
immunosuppression, he has no recent significant history of contact with
resistant organisms and no recent antibiotic therapy, and he is on a very low
amount of immunosuppression. Therefore, answer A is incorrect because you
do not need to cover health-care associated pathogens. Answer B is incorrect
because the patient required hospital admission. Answe C is incorrect
because there is no need to cover healthcare associated pathogens. Answer
D is correct because the patient has community-acquired pneumonia
necessitating hospital admission.
=============================================
45A study found administration of amoxicillin 1-2 hours prior to dental extraction
reduced the presence of bacteria in blood by 46% when blood cultures were
drawn immediately after extraction. Which patient would benefit most from
amoxicillin taken orally before a root canal?
A 46-year-old man with unilateral total knee replacement 6 months
A
ago
A 54-year-old woman with unilateral total hip replacement 5 years
B
ago
C
A 65-year-old man with mitral valve prolapse
A 72-year-old woman with a history of bioprosthetic mitral valve
D
replacement

Answer Explanation:
It was previously thought that those with a history of knee and hip replacement
required antibiotic prophylaxis before dental procedures to prevent hip and
knee infections. However, this is no longer true; therefore, prophylaxis before
dental procedures is no longer recommended. Prophylaxis continues to be
recommended for patients with a history of valve replacement for the
prevention of infective endocarditis, but not for patients with valvulopathy,
including those with valvular prolapse.
46Vignette:
A 37-year-old woman is admitted to the trauma intensive care unit for a
fracture-dislocation of the thoracic spine secondary to a motor vehicle collision.
She is immediately taken to the operating room for stabilization of her spine.
She has no significant medical history and no known drug allergies.
Which option is most appropriate for perioperative antibiotic administration in
this patient?
A
Vancomycin
B
Piperacillin/tazobactam
C
Ceftriaxone
D
Cefazolin
Answer Explanation:
Cefazolin has an appropriate spectrum of activity against gram-positive
organisms such as Staphylococcusspp., adequate tissue penetration, and
adequate concentrations in the spinal disks. Furthermore, the guidelines for
antimicrobial prophylaxis in surgery specify that patients undergoing
orthopedic procedures involving the spinal cord receive prophylaxis with
cefazolin (i.e., first-generation cephalosporin); therefore, answer D is correct.
Vancomycin is an alternative choice for patients with a beta-lactam allergy or
those colonized with MRSA (methicillin-resistant S. aureus); therefore, Answer
A is not the best choice. Using broader coverage promotes bacterial resistance
and is unnecessary in this scenario; therefore, Answer B and Answer C are
also incorrect.
=======================================
47M.J. is an 85-year-old woman whose daughter brings her to the physicians office because the family has
noticed that she has been sleeping more lately and that she seems very confused. On physical examination,
M.J. is lethargic and not alert and oriented. Her vital signs include temperature of 97.5F, blood pressure
88/55 mm Hg, heart rate 90 beats/minute, and respiratory rate 27 breaths/minute. A chest radiograph taken in
the office reveals a left lower lobe consolidation, and she is given a diagnosis of CAP. Using the CURB

point-of-care patient scoring system, which statement is the best recommendation for proceeding with the
treatment of M.J.s CAP?
A. Treat her as an outpatient for 3 days and reassess.
B. Treat her as an outpatient for 14 days and follow up.
C. Transfer her to the emergency department at the local hospital to be admitted for treatment.
D. Immediately have her transferred to the ICU.

Answer is C
CAP
Which means community acquired pneumonia
In CAP
HAP)hospital acquired pneumonia

curb 65 score
5

48-P.E. is a 56-year-old man who comes to the clinic with a 3-day history of fever, chills, pleuritic chest
pain, malaise, and cough productive of sputum. In the clinic, his temperature is 102.1F (38.9C) (all other
vital signs are normal). His chest radiograph shows consolidation in the right lower lobe. His white blood
cell count (WBC) is 14.4/mm3, but all other laboratory values are normal. He is given a diagnosis of
community-acquired pneumonia. He has not received any antibiotics in 5 years and has no chronic disease
states. Which one of the following is the best empiric therapy for P.E.?
A. Doxycycline 100 mg orally 2 times/day.
B. Cefuroxime axetil 250 mg orally 2 times/day.
C. Levofloxacin 750 mg/day orally.
D. Trimethoprim/sulfamethoxazole (TMZ/SMZ) double strength orally 2 times/day
The answer: A
This patient with CAP(community acquired pneumonia)
Calculate Curb 65 score = 0 or max 1 as confusion is not mentioned ,,,,, so oupatient treatment
Following that
We look for comorbidities +/- history of antibiotics administration over the last 3 months
If patient has neither comorbidities nor history of taking antibiotics in tha past 3 months,,,,,, so less risk of
drug resistance so monotherapy
This patient with CAP so the most suspected microorganism is mycoplasma or chlamydia which are atypical
bacteria so the choice will be doxycycline or macrolides
If patient with comorbidities or history of taking antibiotics so the choice will be between fluoroquinolones
only or double therapy and one of them should be doxy or macrolide
In this case patient has no history of AB nor comorbidities so the treatment will be monotherapy macrolide
or doxycyclin
So the preferred answer is A
Moreover

Exclude B as cefuroxime has no role in CAP


Exclude D as sulfa has no role in CAP
Exclude C as patient has neither comorb nor history of taking AB
49R.C. is a 60-year-old woman who presents to the clinic with a 4-day history of increasing productive cough,
malaise, wheezing, and fever. Her medical history includes type 2 diabetes mellitus for 20 years, congestive
heart failure, chronic kidney disease, and osteoarthritis. On examination, she has a temperature of 102.3F,
respiratory rate 22 breaths/minute, blood pressure 120/78 mm Hg, and heart rate 90 beats/minute. She is 56
tall and weighs 90 kg. Her laboratory values are within normal limits except for serum creatinine 3.0 mg/L
and WBC count 18/mm3. A chest radiograph reveals consolidation in the right lower lobe. She is given a
diagnosis of CAP. Which regimen is the best empiric option for managing R.C.s CAP?
A. Levofloxacin 750 mg orally once daily for 10 days.
B. Azithromycin 500 mg orally once on day 1; then 250 mg orally daily for 4 days.
C. Linezolid 600 mg orally twice daily for 10 days.
D. Azithromycin 500 mg orally once on day 1; then 250 mg orally daily for 4 days plus amoxicillin 500 mg
orally twice daily for 10 days
Answer: D
According to the IDSA/ATS guidelines, patients with comorbidities such as diabetes, immunosuppression,
renal failure, and heart failure should be given a respiratory fluoroquinolone (levofloxacin, moxifloxacin,
gemifloxacin) or azithromycin or clarithromycin PLUS high-dose amoxicillin or amoxicillin/clavulanate
because of the risk of drug-resistant S. pneumoniae. This patient has a calculated CrCl less than 30
mL/minute; therefore, levofloxacin 750 mg/day would not be the appropriate dose
EBPLAIN:
this patient had CAP with comorbidities (heart failure,diabetes,osteoarthritis,renal impairement so treatment
will be either respiratory fluoroquinolones as monotherapy or double therapy contain macrolides or
doxycyclin plus high dose amoxicillin so by exclusion ,,exclude b & c
this patient with renal impairement so i should calculate CrCl for this patient ,,,,,,,this done by Cockcraft
&Gault equation
CrCl= (140 - age) x weight / 72 X serum cr
multiply by 0.85 if female
by applying this equation ,,,,,,age=60 ,,,weight =90,,,cr =3 ,,,,,,,it will be 33.3
levofloxacin is mainly renal excreted so i should refer to reference to know the dose of levo in this patient
http://globalrph.com/levofloxacin_renal.htm
dose in this patient should be 750 mg every other day

5047-year-old male is admitted to the general medicine floor of your hospital with a 3-day history of a
productive cough with greenish yellow sputum and shortness of breath. He has a white blood cell count of
13 K/mm3(baseline 5), serum creatinine of 1.2 mg/dL (baseline 11.3 mg/dL), and Tmax of 38.3C, and his

admission chest X-ray shows a left lower lobe pneumonia. He has no known drug allergies but a medical
history significant for a living related kidney transplant 10 years ago for end-stage renal disease caused by
polycystic kidney disease. In his history from a transplant clinic note 5 months ago, you see that he has had
no recent treated rejections; is maintained on tacrolimus 1 mg orally twice daily with goal serum troughs
between 2 and 4 ng/mL and prednisone 5 mg daily; and is seen yearly by the transplant nephrologists but
that he has no other history of hospital or emergency department contact. The medical resident calls you for
help with antibiotic initiation. Question # 138 Which of the following is the best choice for empiric
antibiotic therapy?
A-Piperacillin/tazobactam 4.5 g intravenously every 6 hours and vancomycin dosed for trough 1520
mcg/mL
B-Azithromycin 500 mg orally x 1; then 250 mg orally daily x 4 more days
C-Cefepime 2 g intravenously every 8 hours, tobramycin 7 mg/kg intravenously every 24 hours, and
vancomycin dosed for a trough 1520 mcg/dL
D- Ceftriaxone 1 g intravenously every 24 hours and azithromycin 500 mg orally daily

Answer: D
Although the patient is immunocompromised on immunosuppression, he has no recent significant history of
contact with resistant organisms and no recent antibiotic therapy, and he is on a very low amount of
immunosuppression. Therefore, answer A is incorrect because you do not need to cover health-care
associated pathogens. Answer B is incorrect because the patient required hospital admission. Answe C is
incorrect because there is no need to cover healthcare associated pathogens. Answer D is correct because the
patient has community-acquired pneumonia necessitating hospital admission

51A 2-year-old child has experienced total body hives following amoxicillin administration. Which agent could
be safely administered to this patient?
A. Cefdinir
B. Cefaclor
C. Cephalexin
D. Cephradine
Answer : A
Cefdinir (correct answer, because, according to the American Academy of Pediatrics, it is the only agent
with a dissimilar 7-position side chain of the listed oral cephalosporins. This specific patient is showing
evidence of immunoglobulin E (IgE)-associated reaction (hives), and the other three listed agents are
similarly in the 7-position; thus, they more likely to be associated with cross-reactivity with amoxicillin).
REFERENCE: MOCK 2013

52
A.W. is a 5-year-old boy (27.3 kg) brought to the pediatric clinic with a high temperature, chills, malaise,
sore throat, dry cough, and nausea and diarrhea that started about 1 day ago. A rapid influenza comes back
positive for influenza A. Which is the best therapy recommendation for A.W.?

A. Zanamivir 2 inhalations (5 mg per inhalation) twice daily for 5 days plus acetaminophen for fever.
B. Supportive care only with acetaminophen for fever
. C. Oseltamivir 60 mg orally twice daily for 5 days plus Pepto-Bismol for nausea and diarrhea.
D. Oseltamivir 60 mg orally twice daily for 5 days plus acetaminophen for fever.

Answer: D
This patient presents within 48 hours of symptom onset, so antiviral therapy can be initiated. Although
zanamivir may be used for prevention in those 5 years and older, it is not indicated for treatment of influenza
in patients younger than 7 years. Aspirin or aspirin-containing products (e.g., bismuth
subsalicylatePepto-Bismol) should not be administered to any child or adolescent younger than 19 years
with confirmed or suspected influenza because of the risk of Reye syndrome. For relief of fever, other
antipyretic medications (e.g., acetaminophen or NSAIDs) are recommended. Therefore, oseltamivir 60 mg
orally twice daily for 5 days (dose for children 2440 kg) plus acetaminophen for fever is the best choice
53Community-acquired pneumonia is most commonly associated with
A) Staphylococcus aureus
B) Listeria monocytogenes
C) Legionella species
D) Streptococcus pneumonia
answer : D

54It is important to identify patients likely to have healthcare-associated pneumonia because


A) Empiric therapy is very different compared to hospital-acquired pneumonia.
B) These patients are more likely to receive inappropriate therapy and have a higher risk of mortality.
C) These patients will not require hospitalization
D) Broad-spectrum antibiotics are not routinely recommended for empiric therapy since MDR pathogens are
unlikely.

Answer : B

55Which of the following would be the most appropriate therapy for the treatment of Mycoplasma pneumonia
for a patient with compliance issues and currently receiving theophylline?
A) Erythromycin
B) Azithromycin
C) Clindamycin
D) Clarithromycin

Answer:B
Azithromycin will be preferred in such case for many reasons
It is the least one to cause drug interactions
Also it the least macrolide to cause QT prolongation but erythromycin and clarithromycin cause it more and
here as the pateint is already on theophylline which in turn causes Arrythmia so better to avoid both and use
azithromycin

56A 35-year-old woman is admitted for suspected septic shock secondary to pneumonia. On admission, her
heart rate was 102 beats/minute, and her blood pressure was 80/50 mm Hg after fluid resuscitation. What is
the most appropriate initial resuscitation strategy for this patient?
A. Begin antibiotic therapy within 1 hour.
B. Maintain goal mixed venous O2 saturation greater than 90%.
C. Start intravenous hydrocortisone 100 mg every 8 hours.
D. Give intravenous fluid challenge if urine output is less than 0.5 mL/kg/hour.
Answer :A
Begin antibiotic therapy within 1 hour.
According to the Surviving Sepsis Campaign Guidelines 2008, antibiotics should be administered within
1 hour of recognition of severe sepsis or septic shock as part of the initial resuscitation strategy. A
patients goal mixed venous oxygen saturation should be 65% or more. Although steroids such as
hydrocortisone decrease mortality and provide shock reversal in certain populations, particularly when
blood pressure is unresponsive to vasopressors, corticosteroids did not provide mortality reduction in a
large clinical trial of patients with septic shock in whom blood pressure may or may not have been
responsive to vasopressors (CORTICUS study). Thus, there is no longer a routine recommendation to
initiate hydrocortisone for initial resuscitation of patients with severe sepsis or septic shock.
Hydrocortisone can be initiated (maximum 300 mg/day) in patients whose blood pressure is not
responsive to fluid resuscitation or vasopressor support. Although urine output of less than 0.5
mL/kg/hour for at least 2 hours, despite fluid resuscitation, is a marker of a sepsis diagnosis, it is not part
of the initial sepsis resuscitation strategy to administer intravenous fluids to maintain urine output greater
than 0.5 mL/kg/hour. Fluids are typically administered to maintain a target central venous pressure of 8
mm Hg or more (or 12 mm Hg or more if mechanically ventilated).

57-Vaccine
Pneumococcal vaccinations can be effective at reducing invasive pneumococcal disease.
Important issues regarding the available pneumococcal vaccinations include:
A Using either available pneumococcal vaccination series in children and/or high-risk adults
B Using the pneumococcal polysaccharide (PPSV23) series beginning at 2 months of age
C Using the pneumococcal conjugate (PCV13) series beginning at 2 months of age
D Using the pneumococcal conjugate (PCV13) series in high-risk patients only

Answer: C
According to the recommended vaccination schedule for individuals 018 years (2013), the PCV13 series is
a routine series started at 2 months of age, not just used in high risk individuals making D incorrect. The
PPSV23 series is not routine in early childhood (ie < 2 months of age, making answer B incorrect) and is
reserved for high-risk individuals.


PCV13
4
,,, 15 6 ,,,,,

PPSV23

5 ) (
65 65
65 5
68 5 63

5814- A 6-year-old patient with documented bacterial rhinosinusitis and no known drug allergies is empirically
best-treated with which of the following options:
A. Moxifloxacin
B. Amoxicillin
C. Amoxicillin-clavulanate
D. Azithromycin
Answer:C
Amoxicillin-clavulanate
Amoxicillin/clavulanate (recommended agent in the 2012 Infectious Diseases Society of America [IDSA]
Clinical Practice Guideline for Acute Bacterial Rhinosinusitis in Children and Adults)
59A 78-year-old man is admitted to the general medicine floor with flank pain, altered mental status, and fever.
Urinalysis is significant for white blood cell (WBC) count of 20 cells/high-power field (hpf), large leukocyte
esterase, and nitrite positive. Initial urine Gram stain (clean catch sample) reveals gram-negative rods.
Assuming institutional antibiogram data would not influence selection, what is the best initial therapy?
A. Nitrofurantoin 100 mg orally twice daily
B. Ciprofloxacin 400 mg intravenously twice daily
C. Ampicillin/sulbactam 1.5 g intravenously every 6 hours
D. Piperacillin/tazobactam 3.375 g intravenously every 6 hours

Answer:B
Ciprofloxacin 400 mg intravenously twice daily
Current cystitis/pyelonephritis guidelines recommend a fluoroquinolone (FQ) as first-line therapy for
hospitalized patients with suspected upper tract disease (pyelonephritis). Gram stain reveals gram-negative
rods, increasing the likelihood of Escherichia coli as the causative pathogen. -Lactam therapy is not
preferred unless local susceptibility data reveal significant FQ resistance (greater than 20%) and spectrum of
coverage (specifically for piperacillin/tazobactam is not necessary). Nitrofurantoin is not indicated for upper
urinary tract disease

60The Pharmacy and Therapeutics (P&T) Committee at your hospital recommends that the pharmacy
department conduct a medication-use evaluation on the management of diabetic foot infections. Which
medication regimens would be the best to evaluate for appropriate use in the management of diabetic foot
infections, based on empiric antibiotic regimen and clinical severity of infection?
A. Oral dicloxacillin for mild foot infections
B. Parenteral ceftriaxone for moderate foot infections
C. Oral clindamycin for mild foot infections
D. Parenteral imipenem/cilastatin for moderate foot infections

Answer: D
Parenteral imipenem/cilastatin for moderate foot infections
According to the Infectious Diseases Society of America (IDSA) guidelines on diabetic foot infections,
parenteral imipenem/cilastatin, a broad-spectrum antibiotic, is recommended for use in severe foot infections
when polymicrobial coverage is needed. Therefore, the use of imipenem/cilastatin for diabetic foot
infections of moderate severity would be a preferred target of a medication-use evaluation. Answer D is the
correct answer
61Which vaccine would you recommend to a 6-month-old child?
A. PCV 13
B. PCV 23
C. Influenza (live)
D. Menningococcal
Answr: A
PCV13 replaces a previous conjugate vaccine (PCV7), which protected against 7 pneumococcal types
(rather than the 13 of PCV13) and has been in use since 2000 and is recommended by the Centers of Disease
Control (CDC) for children under 5-years-old
62A patient has confirmed lumbar spinal osteomyelitis and intraoperative cultures reveal
methicillin-susceptible Staphylococcus aureus (MSSA). His past medical

history is significant for long-term depression treated with paroxetine and hypercholesterolemia treated with
simvastatin. Social history is significant for current intravenous drug use. Which antibacterial regimen is
most appropriate in the treatment of this patients infection?
A. Vancomycin
B. Linezolid
C. Cefazolin
D. Telavancin
Answer : C
Cefazolin (correct answer because vancomycin is inferior to cell wallactive antistaphylococcal agents like
cefazolin in the treatment of methicillin-sensitive Staphylococcus aureus [MSSA]; linezolid should be
avoided because of potential serotonin syndrome with concomitant paroxetine; there is no experience with
telavancin in the treatment of osteomyelitis)
63The patient's CSF cultures eventually grow Neisseria meningitidis. Which group of individuals would be
appropriate candidates for chemoprophylaxis?
A: The nurse and physician who examined P.K. when she presented to the family medicine clinic and the
emergency department staff who came in contact with her
B: Her roommate
C: Her roommate, her boyfriend, and the students at the party with whom she may have shared beverages
D: Her boyfriend and her entire dormitory
Answer : C
Chemoprophylaxis should be given to all close contacts within 1 week of symptom onset, which include
those with more than 8 hours of contact within 3 feet of the patient and/or those in contact with oral
secretions. Therefore, her roommate and boyfriend as well as individuals who may have shared drinks with
her at the party, would be indicated.

64A 3-week-old infant presents to the emergency department with a 2-day history of increased fussiness,
decreased oral intake, and vomiting. The lumbar puncture reveals hazy cerebral spinal fluid, white blood
cells (WBCs) of 15,000, and protein of 176. The mothers birth history is unknown. Which is best empiric
antibiotic regimen?
A. Ampicillin plus gentamicin plus acyclovir
B. Ampicillin plus cefotaxime
C. Vancomycin plus gentamicin plus acyclovir
D. Vancomycin plus cefotaxime
Answer : B
Ampicillin plus cefotaxime
The most likely organisms in neonates are group B streptococcus, Escherichia coli, and other
gram-negative enteric bacilli. Although not as common, coverage for listeria monocytogenes should be

part of empiric therapy. Acyclovir is not used empirically unless herpes simplex virus (HSV) is strongly
suggested. Vancomycin is not recommended for initial therapy in a neonate

65All of the following are encapsulated microorganisms except:


A) Streptococcus pneumoniae
B) Listeria monocytogenes
C) Neisseria meningitidis
D) Haemophilus influenzae

Answer : B
listeria monocytogenes

66A 55-year-old woman with a medical history significant for a bioprosthetic mitral valve replacement,
hypertension, and depression has an appointment with the dentist next week to have her teeth cleaned. Her
current medications include aspirin 81 mg orally daily, candesartan 16 mg orally daily, hydrochlorothiazide
25 mg orally daily, and escitalopram 10 mg orally daily. She states that her throat and lips swell when she
takes penicillin.
Which antibiotic regimen, if any, would be most appropriate to prevent infective endocarditis in this patient?
A Amoxicillin 2 g orally 3060 minutes before the procedure
B Azithromycin 500 mg orally 3060 minutes before the procedure
C Clindamycin 600 mg orally 3060 minutes before and after the procedure
D No antibiotic prophylaxis required for this patient

Answer: B
This patient requires antibiotic prophylaxis for infective endocarditis because she has a bioprosthetic mitral
valve and is undergoing a dental procedure (teeth cleaning) that involves manipulation of her gingival tissue.
Therefore, Answer D is incorrect. Only a single dose of antibiotic therapy is required as prophylactic therapy,
with the dose given 3060 minutes before the procedure. Therefore, Answer C is incorrect. This patient
appears to have experienced an anaphylactic reaction to penicillin in the past; therefore, penicillin and
penicillin-like drugs (such as amoxicillin) should be avoided in this patient (Answer A). In patients with an
allergy to penicillin, a single dose of clindamycin, azithromycin, or clarithromycin given 3060 minutes
before the procedure is recommended (Answer B).
67a patient started on 1000mg vancomycin infusion over 60 mints after the start of the iv inf he had red patches
over his body what you should do

1. decrease dose to 500mg


2. stop vanco
3. give the infusion over more than 60 mins
infusion-related events are related to both concentration and rate of administration of vancomycin.
Concentrations of no more than 5 mg/mL and rates of no more than 10 mg/min are recommended in adults.
In selected patients in need of fluid restriction, a concentration up to 10 mg/mL may be used; use of such
higher concentrations may increase the risk of infusion-related events. Infusion-related events may occur,
however, at any rate or concentration.
500 100

5 pneumococcal vaccnation
pcv 13

ppsv23 ()
pcv 13
ppsv23 5
) 5 65 )
pcv13 ppsv 23
ppsv 23 pcv13
ppsv23 pcv13

===================================

68. An elderly diabetic patient is admitted to the hospital with pneumonia. The sputum
culture stains for a gram-negative rod. The patient is started on IV ampicillin. Two days later,
the patient is not improving, and the microbiology laboratory reports the organism to be a
-lactamase producing H. influenzae.
What course of treatment is indicated?
A. Continue with the IV ampicillin.
B. Switch to IV cefotaxime.
C. Switch to oral vancomycin.
D. Add gentamicin to the ampicillin therapy.
Answer: B
-lactamase producing H. influenzae. First line treatment in pneumonia thrd generation
cephalosporin .
===================================

69. A 57-year-old man complains of fever, headache, confusion, aversion to light, and neck
rigidity. A presumptive diagnosis of bacterial meningitis is made. Antimicrobial therapy
should be initiated after which one of the following occurrences?
A. Fever is reduced with antipyretic drugs.
B. Sample of blood and cerebrospinal fluid have been taken.
C. A Gram stain has been performed.
D. The results of antibacterial drug susceptibility tests are available.
E. Infecting organism(s) have been identified by the microbiology laboratory.
Answer: B
===========================
70. In which one of the following clinical situations is the prophylactic use of antibiotics not
warranted?
A. Prevention of meningitis among individuals in close contact with infected patients.
B. Patient with a hip prosthesis who is having a tooth removed.
C. Presurgical treatment for implantation of a hip prosthesis.
D. Patient who complains of frequent respiratory illness.
E. Presurgical treatment in gastrointestinal procedures.
Answer: D
Antibiotic prophylaxis recommendations exist for two groups of patients:
- those with heart conditions that may predispose them to infective endocarditis
- those who have a prosthetic joints and may be at risk for developing hematogenous
infections at the site of the prosthetic.
=============================
71. Which one of the following patients is least likely to require antimicrobial treatment
tailored to the individuals condition?
A. Patient undergoing cancer chemotherapy.
B. Patient with kidney disease.
C. Elderly patient.
D. Patient with hypertension.
E. Patient with liver disease.
Answer: D
============================
72. A 70-year-old alcoholic male with poor dental hygiene is to have his remaining teeth
extracted for subsequent dentures. He has mitral valve stenosis with mild cardiac
insufficiency and is being treated with captopril, digoxin, and furosemide. The dentist

decides that his medical history warrants prophylactic antibiotic therapy prior to the
procedure and prescribes which of the following drugs?
A. Vancomycin.
B. Amoxicillin.
C. Tetracycline.
D. Cotrimoxazole.
E. Imipenem.
Answer: B
drug of choice Amoxicillin given 30-60 m before procedure
=================================
73. Which one of the following is the best route of administration and dosing schedule for
treatment with aminoglycosides based on the drugs concentrationdependent killing
property?
A. Oral every 8 hours.
B. Oral every 24 hours.
C. Parenterally by continuous intravenous infusion.
E 24 8 Because toxicity depends more on duration of
D. Parenterally every 8 hours.
therapeutic levels than on peak levels and because efficacy is
E. Parenterally every 24 hours.
concentration-dependent rather than time-dependent, frequent doses are avoided.
Once/day IV dosing is preferred for most indications except enterococcal endocarditis.
IV aminoglycosides are given slowly (30 min for divided daily dosing or 30 to 45 min for
once/day dosing).

Answer: D
Since they are not absorbed from the gut, they are administered intravenously and
intramuscularly Every 8 hours
==================================
74. C.G. is a 63-year-old woman with a history of breast cancer (treated 5 years ago) and
hypothyroidism. She recently received antibiotics for a lower respiratory infection. Now, she
has had significant, watery diarrhea for the past 3 days. The stool tests positive for
Clostridium difficile toxin. Which one of the following is the best initial antibiotic regimen
for C.G.?
A. Vancomycin 1 g intravenously every 12 hours for 14 days.
B. Vancomycin 125 mg oral 4 times/day for 3 days.
C. Metronidazole 500 mg intravenously every 8 hours for 7 days.
D. Metronidazole 500 mg orally every 8 hours for 10 days.
Answer: D
Metronidazole 500 mg orally every 8 hours for 10 days. Vancomycin given intravenously is
ineffective for Clostridium difficile colitis because not enough drug reaches the infection site.
In addition, the length of therapy is 10-14 days. Vancomycin orally is effective in treating C.
difficile colitis, but a 3-day course is too short to effectively treat the infection. Although
intravenous metronidazole can be used to treat C. difficile colitis, the oral route of

administration is a more optimal regimen. Moreover, 7 days is not long enough to treat the
infection. Therefore, Answer D is the best initial regimen for C.G.
=========================================
75. R.O., a 42-year-old man, presents to the pharmacy for a flu shot. He states that he is in
good health and has no other medical conditions. You note a pack of cigarettes in his shirt
pocket. Which one of the following is the best option for R.O.?
A. Give him the influenza vaccine, and assess his readiness for smoking cessation.
B. Give him the influenza vaccine, and remind him that he will need the pneumococcal
vaccine earlier than age 65 if his smoking causes chronic obstructive pulmonary disease
(COPD).
C. Give him the influenza vaccine, and suggest that he also receive the pneumococcal
vaccine because he is a smoker.
D. Give him the influenza vaccine, and suggest that he, and everyone exposed to his smoking
in the household, receive the pneumococcal vaccine.
Answer: C
Answer C is the best recommendation. Answer A is incorrect. Even though these are
appropriate responses, he also needs to receive the pneumococcal vaccine. Answer B is also
incorrect. Although patients with COPD require the pneumococcal vaccine, R.O. needs the
vaccine now because he is a current smoker. Answer D is incorrect. Although he should
receive the pneumococcal vaccine, household contacts of a smoker need not receive it.
=======================================
76. M.V. is a 48-year-old man with type 2 diabetes mellitus. His diabetes is poorly controlled,
and he now has peripheral neuropathy and early renal insufficiency. He comes to the
emergency department with a red swollen left foot and a deep ulcer that is 2 cm x 3 cm on
the bottom of his left foot. Which one of the following is the best treatment for M.V.?
A. Ertapenem 1 g intravenously daily.
B. Linezolid 600 mg intravenously twice daily.
C. Azithromycin 500 mg intravenously daily.
D. Cefazolin 1 g intravenously every 8 hours.
Answer: A
Answer A is the correct answer. Answer B is incorrect; because M.V.'s foot infection is deep
and potentially limb threatening, he needs to be treated with antibiotics with broad activity.
Linezolid has only gram-positive activity. Azithromycin has very limited activity against
organisms that cause cellulitis and/or diabetic foot infections. Answer D is incorrect; because
M.V.'s foot infection is deep and potentially limb threatening, he needs to be treated with
antibiotics with broader activity.
=======================================

77. W.T. is a 62-year-old woman who presents to her hemodialysis clinic with a 3-day history
of cough, fever, and chills. Yesterday, her temperature was as high as 102.0F, and she began
coughing up yellowish sputum. She has a history of hypothyroidism, hypertension, chronic
renal disease (on hemodialysis 3 days/week), and gastroesophageal reflux disease. Her
medications include levothyroxine, amlodipine, pantoprazole, calcium carbonate,
erythropoietin, and ferrous sulfate. She has had no recent antibiotic therapy. She is given a
diagnosis of pneumonia. Which one of the following would be the best empiric therapy for
W.T.?
A. Doxycycline 100 mg orally twice daily.
B. Levofloxacin 750 mg intravenously x 1; then, 500 mg every 48 hours with dialysis.
C. Piperacillin/tazobactam 2.25 g intravenously every 8 hours plus azithromycin 500 mg
intravenously daily.
D. Cefepime 500 mg intravenously every 24 hours plus tobramycin dosed after each dialysis
plus linezolid 600 mg intravenously every 12 hours.
Answer :D
Doxycycline alone is only recommended in patients with community-acquired pneumonia
(CAP) and no coexisting comorbidities. W.T., because of her history of dialysis, has health
care-associated pneumonia (HCAP). Levofloxacin is appropriate therapy for hospitalized
patients with CAP, but this patient has HCAP. Azithromycin is unnecessary for HCAP
because atypical organisms are uncommon. In addition, this regimen contains only one agent
for Pseudomonas and no antibiotic with methicillin-resistant Staphylococcus aureus (MRSA)
activity. Therefore, Answer D is the best recommendation. A patient with HCAP needs to be
treated for multidrug-resistant organisms - two agents for Pseudomonas and an antibiotic
with MRSA activity.
======================================
78. A 4-year-old boy who presents to the minor medical clinic for treatment of a spider bite
on his leg that is inflamed, red, and draining pus. Which medication is the most appropriate
to empirically cover this patient's methicillin-resistant Staphylococcus aureus (MRSA)
infection?
A. Clindamycin
B. Sulfamethoxazole/trimethoprim (SMX/TMP)
C. Rifampin
D. Doxycycline

Answer: A
Clindamycin has more evidence to support its use than sulfamethoxazole/trimethoprim
(SMX/TMP) and rifampin. Doxycycline should not be used in children younger than 8 years
old.

Clindamycin is an antibiotic of the lincosamide class, which blocks the ribosomes of


microorganisms. It is usually used to treat infections with anaerobic bacteria, but can also be
used to treat protozoal diseases, such as malaria. It is a common topical treatment for acne
and can be useful against some methicillin-resistant Staphylococcus aureus (MRSA)
infections.
============================================
79. Which organism is LEAST likely to contribute to the development of diabetic foot
ulcers?
A. Staphylococcus aureus
B. N. meningitis
C. EnterococciI
D. Corynebacterium spp..
Answer: B
Mild-moderate infections (particularly those that are previously untreated) are commonly
caused by gram- positive organisms (Staphylococcus and Streptococcus). Answer C and
Answer D can be found commonly in polymicrobial infections, which are more severe and
progressive. Answer B is not a common diabetic foot infection
============================================
80. A 31-year-old woman, who is a known intravenous drug abuser, is admitted to the cardiac
care unit with fever and tachycardia. A pair of initial blood cultures are positive for grampositive cocci.
Vancomycin 1gram intravenously every 8 hours and
Piperacillin/Tazobactam 4.5 grams intravenously every 6 hours are initiated pending cultures.
A transthoracic echocardiogram (TTE) shows a 1.1 cm vegetation on the tricuspid valve.
On day 4 cultures and susceptibilities are available showing methicillin- resistant
Staphylococcus aureus with a vancomycin MIC is 1mcg/mL. Additional susceptible agents
include: sulfamethoxazole-trimethoprim, clindamycin, daptomycin and gentamicin. The
patient will be medical management only. What is the best definitivetherapy in this patient?
A. Vancomycin plus gentamicin
B. Vancomycin
C. Daptomycin
D. Daptomycin plus gentamicin
Answer : B
Gentamicin therapy is not indicated in native valve endocarditis due to MRSA.
Vancomycin therapy is appropriate assuming clinical response. MIC of >1mcg/mL have
been associated with unfavorable outcomes, and may suggest alternative therapy; however
vancomycin monotherapy is appropriate in this patient

==============================================
81. Which patient should receive antibiotic prophylaxis prior to dental procedures?
A. Patient a with history of mitral valve prolapse diagnosed 24 years ago.
B. Patient a with history of infective endocarditis diagnosed 4 years ago.
C. Patient a with history of surgically-repaired foramen ovale (PFO) with patch 2 years
ago.
D. Patient a with history of mitral stenosis diagnosed 7 years ago
Answer: B
Guidelines for the prevention of infective endocarditis (IE) were updated in 2007. These
recommendations significantly reduced the population of patients who should be considered
for antibiotic prophylaxis. Patients with a history of valvular abnormalities are not candidates
for antibiotic prophylaxis unless they have a history if IE or recent repair (within 6 months)
or unless a prosthetic heart valve is present. Therefore, the most correct answer is the patient
in Answer B, who has history of developing IE
================================================
82-A pregnant woman was hospitalized and catheterized with a Foley catheter. She
developed a urinary tract infection caused by Pseudomonas aeruginosa and was treated with
gentamicin. Which of the following adverse effects was a risk to the fetus when the woman
was on gentamicin?
A. Skeletal deformity.
B. Hearing loss.
C. Teratogenesis.
D. Blindness.
E. Mental retardation.
Answer: B
================================================
83-All of the following factors may increase the risk of nephrotoxicity from gentamicin
therapy except which one?
(A) age 70 years
(B) prolonged courses of gentamicin therapy
(C) concurrent amphotericin B therapy
(D) trough gentamicin levels 2 mg/mL
(E) concurrent cisplatin therapy
Answer: D
Trough serum levels
2 mg/mL are considered appropriate for gentamicin and are
recommended to minimize the risk of toxicity from this aminoglycoside. Because
aminoglycosides accumulate in the proximal tubule of the kidney, nephrotoxicity can occur.

=====================================
84-BT is a 43-year-old female seen by her primary-care physician for a mild staphylococcal
cellulitis on the arm. Which of the following regimens would be appropriate oral therapy?
(A) dicloxacillin 125 mg every 6 hrs
(B) vancomycin 250 mg every 6 hrs
(C) methicillin 500 mg every 6 hrs
(D) cefazolin 1 g every 8 hrs
(E) penicillin V 500 mg every 6 hrs
Answer: A

Anda mungkin juga menyukai